FAR CPA Exam - Financial Statement Accounts

Ace your homework & exams now with Quizwiz!

Baker Co. issued 100,000 shares of common stock in the current year. On October 1, Baker repurchased 20,000 shares of its common stock on the open market for $50 per share. At that date, the stock's par value was $1 and the average issue price was $40 per share. Baker uses the cost method for treasury stock transactions. On December 1, Baker reissued the stock for $60 per share. What amount should Baker report as treasury stock gain at December 31?

$0 Treasury stock transactions are equity transactions and result in no gain or loss. Treasury stock transactions affect additional paid-in capital accounts and retained earnings.

Lin Co., a distributor of machinery, bought a machine from the manufacturer in November for $10,000. On December 30, Lin sold this machine to Zee Hardware for $15,000 under the following terms: 2% discount if paid within 30 days, 1% discount if paid after 30 days but within 60 days, or payable in full within 90 days if not paid within the discount periods. However, Zee had the right to return this machine to Lin if Zee was unable to resell the machine before expiration of the 90-day payment period, in which case Zee's obligation to Lin would be canceled. In Lin's net sales for the year ended December 31, how much should be included for the sale of this machine to Zee?

$0 When a right to return is allowed, sales revenue is often deferred until the right of return lapses unless all six required conditions are met sooner than that. In this case, the second requirement to recognize the revenue is not yet met—the buyer has to have paid the seller, or the buyer is required to pay the seller, and that obligation is not contingent on the buyer's resale of the item. Here the item can be returned and not need to be paid for, if not resold during the 90-day return period. Thus, the return period has to expire before the revenue can be recognized.

Cobb Co. purchased 10,000 shares (2% ownership) of Roe Co. on February 12, 20X1. Cobb received a stock dividend of 2,000 shares on March 31, 20X1, when the carrying amount per share on Roe's books was $35 and the market value per share was $40. Roe paid a cash dividend of $1.50 per share on September 15, 20X1. In Cobb's income statement for the year ending October 31, 20X1, what amount should Cobb report as dividend income?

$18,000

For its first year of operations, Cable Corp. recorded a $100,000 expense in its tax return that will not be recorded in its accounting records until next year. There were no other differences between its taxable and financial statement income. Cable's effective tax rate for the current year is 45%, but a 40% rate has already been passed into law for next year. In its year-end balance sheet, what amount should Cable report as a deferred tax asset (liability)?

$40,000 liability

On January 1 of the current year, Wren Co. leased a building to Brill under an operating lease for 10 years at $50,000 per year, payable the first day of each lease year. Wren paid $15,000 to a real estate broker as a finder's fee. The building is depreciated $12,000 per year. For the year, Wren incurred insurance and property tax expense totaling $9,000. Wren's net rental income for the year should be:

$50,000 - $12,000 - $9,000 - $1,500 = $27,500

On January 2 of the current year, Cruises, Inc. borrowed $3,000,000 at a rate of 10% for three years and began construction of a cruise ship. The note states that annual payments of principal and interest in the amount of $1,300,000 are due every December 31. Cruises used all proceeds as a down payment for construction of a new cruise ship that is to be delivered two years after start of construction. What should Cruise report as interest expense related to the note in its income statement for the second year?

0 The cruise ship qualifies for interest capitalization. Qualifying assets, per FASB ASC 835-20-15-5, include "assets that are constructed or otherwise produced for an entity's own use (including assets constructed or produced for the entity by others for which deposits or progress payments have been made)."

On day 1, Clothes Co., sells clothing to Link Corp. for $40,000. Clothes ships the clothing on day 1 and Link is obligated to pay Clothes within 6 months. Link is given 12 months to return any of the clothing for a refund if they experience low demand. Link is also given 18 months to exchange any clothing due to low demand. At the time of sale, Clothes cannot reasonably estimate returns, but estimates $5,000 in exchanged goods. Clothes should recognize revenue for the aforementioned transaction:

12 months after the date of sale.

The term "tax position" as used by the FASB refers to which of the following?

A position in a previously filed tax return or a position expected to be taken in a future tax return that is reflected in measuring current or deferred income tax assets and liabilities for interim or annual periods. A tax position can result in a permanent reduction of income taxes payable, a deferral of income taxes otherwise currently payable to future years, or a change in the expected realizability of deferred tax assets. The term tax position also encompasses, but is not limited to: A decision not to file a tax return An allocation or a shift of income between jurisdictions The characterization of income or a decision to exclude reporting taxable income in a tax return A decision to classify a transaction, entity, or other position in a tax return as tax exempt An entity's status, including its status as a pass-through entity or a tax-exempt not-for-profit entity.

What is the present value of all future retirement payments attributed by the pension benefit formula to employee services rendered prior to that date and based on current and past compensation levels?

Accumulated benefit obligation

Of the 125,000 shares of common stock issued by Vey Corp., 25,000 were held as treasury stock on December 31, 20X1. During 20X2, transactions involving Vey's common stock were as follows: January 1 through October 31: 13,000 treasury shares were distributed to officers as part of a stock compensation plan. November 1: A 3-for-1 stock split took effect. December 1: Vey purchased 5,000 of its own shares to discourage an unfriendly takeover. These shares were not retired. On December 31, 20X2, how many shares of Vey's common stock were issued and outstanding?

After 13,000 of the shares are reissued, only the remaining 12,000 (25,000 - 13,000) are treasury stock and the rest of the issued shares (125,000 - 12,000, or 113,000) are outstanding. The stock split affects all outstanding stock. Afterwards, there are 375,000 issued, 36,000 treasury stock (12,000 × 3) and 339,000 outstanding (375,000 - 36,000, or 339,000). The repurchase of the 5,000 shares increases treasury stock to 41,000 (36,000 + 5,000) and decreases outstanding stock to 334,000 (339,000 - 5,000). Issued shares are not affected.

At December 31, 20X1, Bren Co. had the following deferred income tax items: A deferred income tax liability of $15,000 related to a noncurrent asset A deferred income tax asset of $3,000 related to a noncurrent liability A deferred income tax asset of $8,000 related to a current liability Which of the following should Bren report in the noncurrent section of its December 31, 20X1, balance sheet?

Amount to be reported in Bren's noncurrent section of balance sheet at December 31, 20X1: Deferred tax liability $15,000 Less deferred tax asset (3,000) Less deferred tax asset (8,000) -------- Net noncurrent liability to be reported on balance sheet $ 4,000 ========

In 20X1, Chain, Inc., purchased a $1,000,000 life insurance policy on its president, of which Chain is the beneficiary. Information regarding the policy for the year ending December 31, 20X6, follows: Cash surrender value (01/01/X6) $ 87,000 Cash surrender value (12/31/X6) 108,000 Annual advance premium paid (01/01/X6) 40,000 During 20X6, dividends of $6,000 were applied to increase the cash surrender value of the policy. What amount should Chain report as life insurance expense for 20X6?

Annual advance premium payment $40,000 Less increase in cash surrender value ($108,000 - $87,000) 21,000 ------- Life insurance expense for 20X6 $19,000 =======

Under current generally accepted accounting principles, which approach is used to determine income tax expense?

Asset and liability approach

Lake Construction Company has consistently used the percentage-of-completion method of recognizing income. During Year 1, Lake entered into a fixed-price contract to construct an office building for $10,000,000. Information relating to the contract is as follows: At December 31 Year 1 Year 2 Percentage of completion 20% 60% Estimated total cost at completion $7,500,000 $8,000,000 Income recognized (cumulative) 500,000 1,200,000 Contract costs incurred during Year 2 were:

At the end of Year 1, the company was 20% finished with a project costing $7,500,000 total, so they must have already expended $1,500,000 (0.20 × 7,500,000). By the end of Year 2, the company was 60% done with a total estimate of $8,000,000 cost, so by the end of the year, the total costs expended so far must have been $4,800,000 (0.60 × $8,000,000). During Year 2, the additional costs to go from $1,500,000 total cost to $4,800,000 total cost must have been expended, for a total year 2 cost of $3,300,000 ($4,800,000 - $1,500,000).

In its 20X1 income statement, Cere Co. reported income before income taxes of $300,000. Cere estimated that, because of permanent differences, taxable income for 20X1 would be $280,000. During 20X1, Cere made estimated tax payments of $50,000, which were debited to income tax expense. Cere is subject to a 30% tax rate. What amount should Cere report as income tax expense?

Because no temporary differences exist, income tax expense is the same as income tax due on the tax return. 20X1 income Taxable Current tax expense = income x tax rate = $280,000 x 30% = $84,000

Reid Partners, Ltd., which began operations on January 1, 20X1, has elected to use cash-basis accounting for tax purposes and accrual-basis accounting for its financial statements. Reid reported sales of $175,000 and $80,000 in its tax returns for the years ending December 31, 20X2 and 20X1, respectively. Reid reported accounts receivable of $30,000 and $50,000 in its balance sheets as of December 31, 20X2 and 20X1, respectively. What amount should Reid report as sales in its income statement for the year ending December 31, 20X2?

Beginning Ending accounts + Sales - Collections = accounts receivable receivable $50,000 + Sales - $175,000 = $30,000 Sales = $175,000 + $30,000 - $50,000 Sales = $155,000

The following information pertained to Azur Co. for the year: Purchases $102,800 Purchase discounts 10,280 Freight-in 15,420 Freight-out 5,140 Beginning inventory 30,840 Ending inventory 20,560 What amount should Azur report as cost of goods sold for the year?

Beginning inventory $30,840 --------- Plus net purchases Purchases $102,800 Purchase discounts (10,280) Freight-in 15,420 --------- Net purchases 107,940 --------- Goods available for sale 138,780 Less ending inventory (20,560) --------- Cost of goods sold $118,220 =========

Which of the following risks are inherent in an interest rate swap agreement? The risk of exchanging a lower interest rate for a higher interest rate The risk of nonperformance by the counterparty to the agreement

Both I and II

Which of the following uses the straight-line depreciation method?

Both group depreciation and composite depreciation

During January of the current year, Haze Corp. won a litigation award for $15,000 which was tripled to $45,000 to include punitive damages. The defendant, who is financially stable, has appealed only the $30,000 punitive damages. Haze was awarded $50,000 in an unrelated suit it filed, which is being appealed by the defendant. Counsel is unable to estimate the outcome of these appeals. In its current year financial statements, Haze should report what amount of pretax gain?

Both of these lawsuits are gain contingencies, which are generally not recognized until it is virtually certain that there are rights to receive the amounts involved. Thus, the amounts that are yet to be received, and subject to the uncertainty of an appeal process, should not be recognized yet. However, the $15,000 award that has not been appealed can be taken now as pretax gain.

During 20X1, Kam Co. began offering its goods to selected retailers on a consignment basis. The following information was derived from Kam's 20X1 accounting records: Beginning inventory $122,000 Purchases 540,000 Freight-in 10,000 Transportation to consignees 5,000 Freight-out 35,000 Ending inventory Held by Kam 145,000 Held by consignees 20,000 In its 20X1 income statement, what amount should Kam report as cost of goods sold?

Cost of goods sold would include all inventoriable costs less ending inventory. Beginning inventory $122,000 Purchases 540,000 Freight-in 10,000 -------- Cost of goods acquired 672,000 Add transportation to consignees 5,000 ------- Cost of goods available $677,000 Less ending inventory Held by Kam $145,000 Held by consignees 20,000 165,000 -------- ------- Cost of goods sold $512,000

On December 30, 20X1, Astor Corp. sold merchandise for $75,000 to Day Co. The terms of the sale were net 30, Free on Board shipping point. The merchandise was shipped on December 31, 20X1, and arrived at Day on January 5, 20X2. Due to a clerical error, the sale was not recorded until January 20X2 and the merchandise, sold at a 25% markup, was included in Astor's inventory on December 31, 20X1. As a result, Astor's cost of goods sold for the year ending December 31, 20X1, was:

Cost of merchandise = Sales price / Markup + 100% = $75,000 / 1.25 = $60,000

In January 20X1, Vorst Co. purchased a mineral mine for $2,640,000 with removable ore estimated at 1,200,000 tons. Vorst believes it will be able to sell the property afterwards for $300,000. During 20X1, Vorst incurred $540,000 of development costs preparing the mine for production and removed and sold 60,000 tons of ore. In its 20X1 income statement, what amount should Vorst report as depletion?

Cost of ore deposit: Purchase price $2,640,000 Development cost 540,000 ---------- Subtotal 3,180,000 Less expected disposal value 300,000 --------- Net cost of ore deposit $2,880,000 Depletion cost per ton = $2,880,000 / 1,200,000 tons = $2.40/ton 20X1 depletion = 60,000 tons x $2.40 = $144,000

Which of the following statements is correct concerning start-up costs?

Costs of start-up activities, including organization costs, should be expensed as incurred.

Venus Corp.'s worksheet for calculating deferred income taxes for 20X1 follows: 20X1 20X2 20X3 ------- ------- ------- Pretax income $1,400 0 0 Temporary differences: Depreciation (800) (1,200) $2,000 Warranty costs 400 (100) (300) ------- ------- ------- Taxable income $1,000 (1,300) 1,700 Loss carryback (1,000) 1,000 Loss carryforward 300 (300) ------- ------- ------- $ 0 $ 0 $1,400 ======= ======= ======= Enacted rate 30% 30% 25% Deferred tax liability (asset): Noncurrent $ 50 ======= Venus had no prior deferred tax balances. In its 20X1 income statement, what amount should Venus report as current income tax expense?

Current income tax expense = 30% × $1,000 = $300, and is defined in FASB ASC 740-10-20 as "the amount of income taxes paid or payable (or refundable) for a year is determined by applying the provisions of the enacted tax law to the taxable income or excess of deductions over revenues for that year."

On which of the following dates is a public entity required to measure the cost of employee services in exchange for an award of equity interests, based on the fair market value of the award?

Date of grant

Zeff Co. prepared the following reconciliation of its pretax financial statement income to taxable income for the year ended December 31, 20X1, its first year of operations: Pre-tax financial income $160,000 Nontaxable interest received on municipal securities (5,000) Long-term loss accrual in excess of deductible amount 10,000 Depreciation in excess of financial statement amount (25,000) --------- Taxable income $140,000 ========= Zeff's average tax rate for 20X1 is 27% and the future average tax rate is estimated to be 30%. In its December 31, 20X1, balance sheet, what should Zeff report as deferred income tax liability?

Deferred income tax liability = Temporary differences x Tax rate = ($25,000 - $10,000) x 0.30 = $15,000 x 0.30 = $4,500

Black Co., organized on January 2, Year 1, had pretax financial statement income of $500,000 and taxable income of $800,000 for the year ended December 31, Year 1. The only temporary differences are accrued product warranty costs, which Black expects to pay as follows: Year 2 $100,000 Year 3 50,000 Year 4 50,000 Year 5 100,000 The enacted income tax rates are 25% for Year 1, 30% for Years 2 through 4, and 35% for Year 5. Black believes that future years' operations will produce profits. In its December 31, Year 1, balance sheet, what amount should Black report as deferred tax asset?

Deferred tax assets are measured by the total temporary differences multiplied by the tax rates in effect when the tax differences unwind. All deferred tax liabilities and deferred tax assets are classified on the balance sheet as noncurrent. The total temporary differences are the $300,000 of warranty costs, which will lower future taxable income. The time when the differences unwind is Years 2 through 5, and the tax asset is measured by the amounts and the rates for these years. The Year 2 amount is $100,000 × 0.30 (30%) = $30,000. For Year 3, the amount is $50,000 × 0.30 = $15,000. Year 4: $50,000 × 0.30 = $15,000 Year 5: $100,000 × 0.35 = $35,000 The total of these amounts is the total deferred tax asset of $95,000: $30,000 + $15,000 + $15,000 + $35,000 = $95,000

Dodd Corp. is preparing its December 31 current-year financial statements and must determine the proper accounting treatment for the following situations: For the current year ended December 31, Dodd has a loss carryforward of $180,000 available to offset future taxable income. However, there are no temporary differences. Based on an analysis of both positive and negative evidence, Dodd has reason to believe it is more likely than not that the benefits of the entire loss carryforward will be realized within the carryforward period. On 12/31 of this year, Dodd received a $200,000 offer for its patent. Dodd's management is considering whether to sell the patent. The offer expires on 2/28 of next year. The patent has a carrying amount of $100,000 at 12/31. Assume a current and future income tax rate of 30%. In its current-year income statement, Dodd should recognize an increase in net income of:

Deferred tax assets are measured by the total temporary differences multiplied by the tax rates in effect when the tax differences unwind. The loss carryforward is recognized as a deferred tax asset at the total future deductible amount multiplied by the future tax rate that will be available for the later tax deductions. All deferred tax liabilities and deferred tax assets are classified on the balance sheet as noncurrent. Thus, the deferred tax asset is a tax benefit (lowering of this year's income tax expense) and will increase net income by the total amount of the expected benefit amount of $54,000 ($180,000 deduction × 0.30 (the future tax rate of 30%)). The other gain is not recognized until the sale is finalized and agreed to by both parties.

In Year 1, Lobo Corp. reported for financial statement purposes the following revenue and expenses that were not included in taxable income: Premiums on officers' life insurance under which the corporation is the beneficiary $ 5,000 Interest revenue on qualified state or municipal bonds 10,000 Depreciation deducted for income tax purposes in excess on depreciation reported for financial statement purposes 10,000 Estimated future warranty costs to be paid in Year 2 and Year 3 60,000 Lobo's enacted tax rate for the current and future years is 30%. Lobo expects to operate profitably in the future. There were no temporary differences in prior years. The deferred tax benefit is:

Deferred tax benefits only come about from temporary differences, like depreciation and warranty costs. All deferred tax assets and liabilities are classified as noncurrent, and apply to both financial accounting and to tax accounting, but are taken at different times for each. The other items are only taken into account in financial accounting, and are not income or expense items for taxes. When depreciation for tax purposes is in excess of depreciation for financial accounting, then it will not give rise to a benefit, but instead to a liability. This leaves only the warranty costs, which do give rise to a deferred tax benefit, since the warranty costs will defer to future years' additional tax deductions. The way to measure the deferred tax benefit is by multiplying the estimated warranty costs by the future tax rate of 30%: $60,000 × 0.30 = $18,000

Mill, which began operations on January 1, Year 1, recognizes income from long-term construction contracts under the percentage-of-completion method in its financial statements and under the completed-contract method for income tax reporting. Income under each method follows: Year Completed Contract Percentage of Completion 1 $ -- $300,000 2 400,000 600,000 3 700,000 850,000 The income tax rate was 30% for Years 1 through 3. For years after Year 3, the enacted tax rate is 25%. There are no other temporary differences. Assuming that Mill does not expect any tax losses in the near future, Mill should report in its December 31, Year 3, balance sheet, a deferred income tax liability of:

Deferred tax liabilities are measured by the total temporary differences multiplied by the tax rates in effect when the tax differences unwind. When income is recognized for financial accounting before the time it is recognized for tax purposes, then it will give rise to a deferred tax liability. All deferred tax liabilities and deferred tax assets are classified on the balance sheet as noncurrent. Here, this company is recognizing the contract income for financial accounting using the percentage-of-completion method, and thus more income is recognized sooner than for taxes, where it uses the slower completed-contract method. The total amount of deferred income is $650,000, which is the additional income under percentage of completion over completed contract ($300,000 + $600,000 + $850,000 - $400,000 - $700,000). This deferred tax liability is measured using the enacted future tax rate of 25%, thus it is $162,500 ($650,000 × 0.25).

A deferred tax liability may result from which of the following items?

Depreciation of tangible assets

Rye Co. purchased a machine with a 4-year estimated useful life and an estimated 10% salvage value for $80,000 on January 1, 20X0. In its income statement, what would Rye report as the depreciation expense for 20X2 using the double-declining-balance method?

Double-declining-balance rate = 2 (1/4 years) = .50/year 20X0 DDB depre. = .50 ($80,000) = $40,000 20X1 DDB depre. = .50 ($80,000-$40,000) = $20,000 20X2 DDB depre. = .50 ($80,000-$40,000-$20,000) = $10,000

During 20X1, Brad Co. issued 5,000 shares of $100 par convertible preferred stock for $110 per share. One share of preferred stock can be converted into three shares of Brad's $25 par common stock at the option of the preferred shareholder. On December 31, 20X2, when the market value of the common stock was $40 per share, all of the preferred stock was converted. What amount should Brad credit to common stock and to additional paid-in capital common stock as a result of the conversion?

During 20X1, Brad Co. issued 5,000 shares of $100 par convertible preferred stock for $110 per share. One share of preferred stock can be converted into three shares of Brad's $25 par common stock at the option of the preferred shareholder. On December 31, 20X2, when the market value of the common stock was $40 per share, all of the preferred stock was converted. What amount should Brad credit to common stock and to additional paid-in capital common stock as a result of the conversion?

On January 2, 20X1, East Corp. adopted a defined benefit pension plan. The plan's service cost of $150,000 was fully funded at the end of 20X1. Prior service cost was funded by a contribution of $60,000 in 20X1. Amortization of prior service cost was $24,000 for 20X1. Assuming that no amortization of unrecognized gain or loss is required in 20X1, what amount should East recognize as pension expense in 20X1?

East Corp.'s pension expense for 20X1 is $174,000, as shown below: Service cost $150,000 Interest cost 0 Amortization of past service cost 24,000 Expected return on plan assets 0 Amortization of unrecognized gain/loss 0 -------- $174,000 ========

Ross Co. pays all salaried employees on a Monday for the 5-day workweek ended the previous Friday. The last payroll recorded for the year ended December 31, 20X1, was for the week ended December 26, 20X1. The payroll for the week ended January 2, 20X2, included regular weekly salaries of $80,000 and vacation pay of $25,000 for vacation time earned in 20X1 not taken by December 31, 20X1. Ross had accrued a liability of $20,000 for vacation pay at December 31, 20X0. In its December 31, 20X1, balance sheet, what amount should Ross report as accrued salary and vacation pay?

Employees were paid through Friday, December 26, 20X1. Salaries were unpaid for Monday, December 29 through Wednesday, December 31 (three days). Accrued salary at 12/31/X1 (3/5 x $80,000) $48,000 Accrued vacation pay for 20X1 25,000 ------- Total Accrual at 12/31/X1 $73,000 =======

Which of the following circumstances would result in a deferred tax asset for the current year?

Expenses that are recognized in financial income this year and deductible next year

NuCorp. agreed to give Rand Co. a machine in full settlement of a note payable to Rand. The machine's original cost was $140,000. The note's face amount was $110,000. On the date of the agreement: the note's carrying amount was $105,000, and its present value was $96,000. the machine's carrying amount was $109,000, and its fair value was $96,000. Assuming that this trade was made as part of troubled debt restructuring, what amount of gains/losses should NuCorp. recognize, and how should these be classified in its income statement?

Extraordinary gain/loss: $0; Other gain/loss: $(4,000)

North Corp. has an employee benefit plan for compensated absences that gives employees 10 paid vacation days and 10 paid sick days. Both vacation and sick days can be carried over indefinitely. Employees can elect to receive payment in lieu of vacation days; however, no payment is given for sick days not taken. At December 31, 20X1, North's unadjusted balance of liability for compensated absences was $21,000. North estimated that there were 150 vacation days and 75 sick days available at December 31, 20X1. North's employees earn an average of $100 per day. In its December 31, 20X1, balance sheet, what amount of liability for compensated absences is North required to report?

FASB ASC 710-10-25-7 provides that "an employer is not required to accrue a liability for nonvesting accumulating rights to receive sick pay benefits..." Thus, North Corp.'s liability for compensated absences at December 31, 20X1, is $15,000 for the 150 vacation days (at $100 per day).

Which of the following disclosures is not required of companies with a defined-benefit pension plan?

FASB ASC 715-20-50-1 requires that the following items be disclosed: A description of the plan's key elements, such as investment policies Components of pension expense Reconciliation of projected benefit obligation and fair value of plan assets Funded status Rates used in measuring benefit amounts (discount, return on plan assets, compensation) Best estimates of next year's contributions to the plan FASB ASC 715-20-50-1 also requires the following additional disclosures: For each annual statement of income presented, the amounts recognized in other comprehensive income, showing separately the net gain or loss and net prior service cost or credit. Those amounts shall be separated into amounts arising during the period and reclassification adjustments of other comprehensive income as a result of being recognized as components of net period benefit cost for the period. For each annual statement of income presented, the net transition asset or obligation recognized as a reclassification adjustment of other comprehensive income as a result of being recognized as components of net periodic benefit cost for the period. For each annual statement of financial position presented, the amounts in accumulated other comprehensive income that have not yet been recognized as components of net periodic benefit costs, showing separately the net gain or loss, net prior service cost or credit, and net transition asset or obligation. The amounts in accumulated other comprehensive income expected to be recognized as components of net periodic benefit cost over the fiscal year that follows the most recent annual statement of financial position presented, showing separately the net gain or loss, net prior service cost or credit, and net transition asset or obligation. The amount and timing of any plan assets expected to be returned to the business entity during the 12-month period, or operating cycle if longer, that follows the most recent statement of financial position presented. Thus, the only answer choice that is not required is the overall description of the plan.

On January 2, 20X1, Farm Co. granted an employee an option to purchase 1,000 shares of Farm's common stock at $40 per share. The option became exercisable on December 31, 20X2, after the employee had completed two years of service, and was exercised on that date. The fair value of the option on January 2, 20X1, was $10 per option. What amount should Farm recognize as compensation expense for 20X1?

FASB ASC 718-10-30-2 requires that the fair value method be used. Except in very rare circumstances, the intrinsic value method is no longer acceptable. Accordingly, total compensation cost for Farm Co. is the $10,000 fair value of the options on the grant date (1,000 shares × $10 fair value per option on January 2, 20X1). The $10,000 total compensation cost should be amortized over Farm's 2-year service period at the rate of $5,000 per year ($10,000 ÷ 2 years = $5,000). The compensation expense for 20X1 therefore is $5,000.

Wolf Co.'s grant of 30,000 stock appreciation rights enables key employees to receive cash equal to the difference between $20 and the market price of the stock on the date each right is exercised. The service period is 20X1 through 20X3, and the rights are exercisable in 20X4 and the following year. The market price of the stock was $25 and $28 on December 31, 20X1 and 20X2, respectively. Assuming that the fair value of the stock appreciation rights was $5 at December 31, 20X1, and $8 at December 31, 20X2, what amount should Wolf report as the liability under the stock appreciation rights plan in its December 31, 20X2, balance sheet?

Fair value of SARs at 12/31/X2 (30,000 x $8 fair value) $240,000 Percentage to service period through 12/31/X2 2/3 ------- Liability at 12/31/X2 $160,000 =======

The following pertains to Pell Co.'s construction jobs, which commenced during 20X1: Project 1 Project 2 --------- --------- Contract price $420,000 $300,000 Costs incurred during 20X1 240,000 280,000 Estimated costs to complete 120,000 40,000 Billed to customers during 20X1 150,000 270,000 Received from customers during 20X1 90,000 250,000 If Pell used the percentage-of-completion method, what amount of gross profit (loss) would Pell report in its 20X1 income statement?

For Project 1: Percentage Completed = 20X1 costs / estimated total costs = $240,000 / ($240,000 + $120,000) = 2/3 Contract price recognized in 20X1 (2/3 x $420,000) $280,000 Less 20X1 cost incurred 240,000 -------- Gross profit from Project 1 $ 40,000 For Project 2: Contract price $300,000 Less actual and estimated costs to complete ($280,000 + $40,000) 320,000 -------- Estimated loss (20,000) -------- Pell's total 20X1 gross profit (loss) under percentage-of-completion $ 20,000 ======= Because Project 2 reflects an anticipated loss, the entire amount of the loss (not the percent completion of loss) must be recognized in accordance with the conservatism principle.

On January 2, 20X1, Blake Co. sold a used machine to Cooper, Inc., for $900,000, resulting in a gain of $270,000. On that date, Cooper paid $150,000 cash and signed a $750,000 note bearing interest at 10%. The note was payable in three annual installments of $250,000 beginning January 2, 20X2. Blake appropriately accounted for the sale under the installment method. Cooper made a timely payment of the first installment on January 2, 20X2, of $325,000 which included accrued interest of $75,000. What amount of deferred gross profit should Blake report on December 31, 20X2?

For installment sales, one must compute the gross profit percentage as a percentage of installment sales revenue for each year. The deferred gross profit for a year is the gross profit percentage for the year multiplied by the remaining uncollected installment sale receivables for the year. Gross profit rate = $270,000 ÷ $900,000 = 30% Cash collected prior to December 31, 20X2: January 2, 20X1 $150,000 January 2, 20X2 250,000 -------- Total $400,000 ======== Cash remaining to be collected = $900,000 - $400,000 = $500,000 Deferred gross profit to be reported on December 31, 20X2 = 30% × $500,000 = $150,000

On December 31, Year 1, Moon, Inc., authorized Luna Co. to operate as a franchisee for an initial franchise fee of $100,000. Luna paid $40,000 on signing the agreement and signed an interest-free note to pay the balance in three annual installments of $20,000 each, beginning December 31, Year 2. On December 31, Year 1, the present value of the note, appropriately discounted, is $48,000. Services for the initial fee will be performed in Year 2. In its December 31, Year 1, balance sheet, what amount should Moon report as unearned franchise fees?

Franchise fee revenue is recognized when all material services or conditions relating to the sale have been substantially performed or satisfied by the franchisor. In this case, the services have not been performed. Consequently, the initial fee is still unearned. Paid at signing $40,000 Present value of note 48,000 ------- Initial and unearned franchise fee $88,000

Lang Co. uses the installment method of revenue recognition. The following data pertain to Lang's installment sales for the years ending December 31, 20X1 and 20X2: 20X1 20X2 ------- ------- Installment receivables at year-end on 20X1 sales $60,000 $30,000 Installment receivables at year-end on 20X2 sales - 69,000 Installment sales 80,000 90,000 Cost of sales 40,000 60,000 What amount should Lang report as deferred gross profit in its December 31, 20X2, balance sheet?

Gross profit rates: 20X1 = ($80,000 - $40,000) / $80,000 = 50% 20X2 = ($90,000 - $60,000) / $90,000 = 33.33% Deferred gross profit on December 31, 20X2: On 20X1 sales = 50% x $30,000 = $15,000 On 20X2 sales = 33.33% x $69,000 = $23,000 ------- Total deferred gross profit $38,000

The following is the stockholders' equity section of Harbor Co.'s balance sheet at December 31:

Harbor's book value per share of common stock is $49: Book value of corporation $2,200,000 Divided by shares of stock outstanding (50,000 - 5,000) / 45,000 ---------- Book value per share $ 49 (rounded)

Amar Farms produced 300,000 pounds of cotton during the Year 1 season. Amar sells all of its cotton to Brye Co., which has agreed to purchase Amar's entire production at the prevailing market price. Recent legislation assures that the market price will not fall below $0.70 per pound during the next two years. Amar's costs of selling and distributing the cotton are immaterial and can be reasonably estimated. Amar reports its inventory at expected exit value. During Year 1, Amar sold and delivered to Brye 200,000 pounds at the market price of $.70. Amar sold the remaining 100,000 pounds during Year 2 at the market price of $0.72. What amount of revenue should Amar recognize in Year 1?

Here the available produced finished inventory has a buyer already under contract obligation to buy, and a guaranteed minimum price, so revenue recognition upon production is appropriate. The inventory can be carried at net realizable value, and the sales price can be recognized as revenue in Year 1, upon production, of $210,000 (300,000 pounds × $0.70 a pound, the set minimum price for Year 1).

Which of the following should be disclosed by a company providing health care benefits to its retirees? The assumed health care cost trend rate used to measure the expected cost of benefits to its retirees The accumulated postretirement benefit obligation

I and II

Orleans Co., a cash-basis taxpayer, prepares accrual-basis financial statements. In its current-year balance sheet, Orleans' deferred income tax liabilities increased compared to the previous year. Which of the following changes would cause this increase in deferred income tax liabilities? An increase in prepaid insurance An increase in rent receivable An increase in warranty obligations

I and II Deferred income tax liabilities are caused by items that defer payment of taxes, which cause more taxes to be paid in later years than the income tax expense taken currently. An increase in prepaid insurance can lower taxes now by adding to the expenses deductible, and cause deferral of taxes to the future, so it would qualify a change that would increase deferred tax liabilities. An increase in rent receivable, a pushing forward of the receipt of the rent in cash (when it will be taxed), can also defer taxes to the future and add to later taxes due, so it would also increase deferred tax liabilities. An increase in warranty obligations means one is pushing forward the paying of the expense in cash (which allows the deduction), and this would lower taxes in the future, not add to the future liabilities. All deferred tax assets and liabilities are classified as noncurrent on the balance sheet.

Which of the following should be disclosed in a company's financial statements related to deferred taxes? The types and amounts of existing temporary differences The types and amounts of existing permanent differences The nature and amount of each type of operating loss and tax credit carryforward

I and III only

Larkin Co. reported a taxable loss of $10,000 in 20X1, its first year of operations, and taxable income of $0 in 20X2. Larkin had no temporary or permanent differences in either 20X1 or 20X2. At the end of 20X1 Larkin believed that 30% of the operating loss carryforward would not be realized; therefore, a valuation allowance of $1,200 (30% of $10,000 NOL × 40% tax rate) was necessary. At the end of 20X2, Larkin believes that the valuation allowance is no longer necessary. Assuming a tax rate of 40%, Larkin should report total income tax expense (benefit) in 20X1 and 20X2 of:

In 20X1, Larkin should recognize a noncurrent deferred tax asset and the related noncurrent deferred tax benefit of $4,000 ($10,000 NOL × 40% tax rate). However, Larkin also must recognize a $1,200 valuation allowance in 20X1. Thus, in 20X1 Larkin should recognize a net tax expense (benefit) of $(4,000) + $1,200 = $(2,800). Note that the recognition of the valuation allowance reduces the net tax benefit recognized in 20X1. The decision in 20X2 that the valuation allowance is no longer necessary means that the valuation allowance should be eliminated, as shown in the following entry in 20X2: Valuation allowance 1,200 Tax expense/benefit - deferred 1,200 Therefore, tax expense (benefit) in 20X2 has a credit balance of $(1,200), indicating a deferred tax benefit. This $(1,200) tax benefit recognized in 20X2 is the change in deferred tax expense/benefit arising from changed circumstances causing a change in judgment as to the amount of valuation.

Ichor Co. reported equipment with an original cost of $379,000 and $344,000, and accumulated depreciation of $153,000 and $128,000, respectively, in its comparative financial statements for the years ending December 31, 20X2 and 20X1. During 20X2, Ichor purchased equipment costing $50,000, and sold equipment with a carrying value of $9,000. What amount should Ichor report as depreciation expense for 20X2?

In the context of this problem accumulated depreciation is affected by the asset disposal when the carrying value of the asset sold is written off and by depreciation expense for the current period. These two items account for the net increase of $25,000 ($153,000 - $128,000) in the credit balance of the accumulated depreciation account. The debit change in accumulated depreciation caused by the asset disposal needs to be determined from the facts provided. The equipment account had a beginning balance of $344,000. The $50,000 purchase of new equipment would cause this balance to increase to $394,000. However, the ending balance was $379,000. The only other transaction affecting the equipment account was the disposal of a piece of equipment. Therefore, the original cost of the disposed equipment was $15,000 ($394,000 - $379,000). Since the disposed equipment had a cost of $15,000 and a carrying value of $9,000 (carrying value = cost - accumulated depreciation), the accumulated depreciation associated with the disposed equipment was $6,000 ($9,000 = $15,000 - accumulated depreciation). The beginning credit balance in the accumulated depreciation control account was $128,000. It would have been decreased (debited) for the $6,000 of accumulated depreciation related to the disposed equipment. That would leave a credit balance of $122,000. However, the ending balance was a credit of $153,000. Depreciation expense for the period would also change (increase or credit) the balance of accumulated depreciation. Since the ending balance was $153,000, and the balance without the effect of depreciation expense was $122,000, the depreciation expense must have been $31,000 ($153,000 - $122,000).

In Year 2, Ajax, Inc., reported taxable income of $400,000 and pretax financial statement income of $300,000. The difference resulted from $60,000 of nondeductible premiums on Ajax's officers' life insurance and $40,000 of rental income received in advance. Rental income is taxable when received. Ajax's effective tax rate is 30%. In its Year 2 income statement, what amount should Ajax report as income tax expense—current portion?

Income tax expense—current is the tax currently payable ($400,000 × 0.30 = $120,000).

On May 1 of the current year, Marno County issued property tax assessments for the fiscal year ending the following June 30. The first of two equal installments was due on November 1 of this year. On September 1, Dyur Co. purchased a 4-year-old factory in Marno subject to an allowance for accrued taxes. Dyur did not record the entire year's property tax obligation, but instead records tax expenses at the end of each month by adjusting prepaid property taxes or property taxes payable, as appropriate. The recording of the November 1, payment by Dyur should have been allocated between an increase in prepaid property taxes and a decrease in property taxes payable in which of the following percentages?

Increase in prepaid property taxes, 33-1/3%; Decrease in property taxes payable, 66-2/3% The payment of property taxes, when made, will be a payment covering 6 months' accrual. The payment will be made in the middle of the 6-month period covered, and thus some of the expense will have already accrued. The 6 months covered by the payment are July, August, September, October, November, and December, and the payment is made on November 1. Thus, the payment is two-thirds (4 out of 6 months) for property tax expenses already accrued and payable (lowering the payable) and one-third for expenses yet to accrue (2 out of 6 months, prepaid for November and December).

In its cash flow statement for the current year, Ness Co. reported cash paid for interest of $70,000. Ness did not capitalize any interest during the current year. Changes occurred in several balance sheet accounts as follows: Accrued interest payable $17,000 decrease Prepaid interest 23,000 decrease In its income statement for the current year, what amount should Ness report as interest expense?

Interest expense = Cash interest + Decrease in prepaid interest - Decrease in interest payable Interest expense = $70,000 + 23,000 - $17,000 = $76,000

House Publishers offered a contest in which the winner would receive $1,000,000 payable over 20 years. On December 31, 20X1, House announced the winner of the contest and signed a note payable to the winner for $1,000,000, payable in $50,000 installments every January 2. Also on December 31, 20X1, House purchased an annuity for $418,250 to provide the $950,000 prize monies remaining after the first $50,000 installment, which was paid on January 2, 20X2. In its 20X1 income statement, what should House report as contest prize expense?

January 2, 20X2, installment $ 50,000 Amount required to meet future installment requirements (i.e., present value of 19 annual payments of $50,000) 418,250 -------- Total $468,250

On January 1, 20X1, Card Corp. signed a 3-year, noncancelable purchase contract, which allows Card to purchase up to 500,000 units of a computer part annually from Hart Supply Co. at $.10 per unit and guarantees a minimum annual purchase of 100,000 units. During 20X1, the part unexpectedly became obsolete. Card had 250,000 units of this inventory at December 31, 20X1, and believes these parts can be sold as scrap for $.02 per unit. What amount of probable loss from the purchase commitment should Card report in its 20X1 income statement?

Minimum purchase commitment for 20X2 and 20X3 (100,000 units x $.10/u x 2 years) $20,000 Less scrap recovery (100,000 units x $.02 x 2 years) 4,000 ------- Probable loss from purchase commitment $16,000 =======

Miro Co. began business on January 2, 20X0. Miro used the double-declining-balance method of depreciation for financial statement purposes for its building, and the straight-line method for income taxes. On January 16, 20X2, Miro elected to switch to the straight-line method for both financial statement and tax purposes. The building cost $240,000 in 20X0, which has an estimated useful life of 15 years and no salvage value. Data related to the building is as follows: Accelerated Straight-Line Year Depreciation Depreciation ---- ------------- -------------- 20X0 $30,000 $16,000 20X1 20,000 16,000 Miro's tax rate is 40%. Which of the following statements is correct?

Miro's deferred tax asset should be reduced by $554 in 20X2. During 20X0 and 20X1, the amount of taxes paid was higher than justified by its GAAP income. The amount of the "overpayment" equals the temporary difference in the GAAP basis of the asset ($240,000 - $50,000, or $190,000) and its tax basis ($240,000 - $32,000, or $208,000) at December 31, 20X1, times the 40% tax rate in effect at December 31, 20X1. More taxes have been paid than justified by GAAP income; therefore, the company has an asset for this "prepayment." The asset is called a deferred tax asset. The temporary difference is $18,000, making the deferred tax asset at December 31, 20X1, $18,000 × 40%, or $7,200. Changes in depreciation methods are accounted for prospectively. Therefore, the change to the straight-line method for GAAP purposes does not eliminate the temporary difference. However, straight-line depreciation for GAAP purposes in 20X2 will be less than that for tax purposes. GAAP depreciation is $190,000 ÷ 13 years, or $14,615. The GAAP basis of the asset at December 31, 20X2, is $240,000 - $64,615, or $175,385; the tax basis is $240,000 - $48,000, or $192,000. The temporary difference is reduced to $16,615. The deferred tax asset at December 31, 20X2, is $16,615 × 40%, or $6,646. Therefore, the deferred tax asset balance decreased by $554 during 20X2. Straight- GAAP Line Tax Deferred Depreciation Depreciation Difference Rate Tax ------------ ------------ ---------- ---- -------- 20X0 $30,000 - $16,000 = $14,000 x 0.40 = $5,600 20X1 20,000 - 16,000 = 4,000 x 0.40 = 1,600 20X2 14,615 - 16,000 = -1,385 x 0.40 = -554 ------- ------- ------- ------ $64,615 $48,000 $16,615 $6,646 ======= ======= ======= ====== ($240,000 - $20,000 - $30,000) ÷ 13 = $14,615 $16,000 - $14,615 = $1,385 $1,385 × 0.40 = $554

When the equity method is used to account for investments in common stock, which of the following affects the investor's reported investment income?

Neither a change in market value of the investee's common stock nor cash dividends from the investee

Inge Co. determined that the net value of its accounts receivable on December 31, 20X1, based on an aging of the receivables, was $325,000. Additional information is as follows: Allowances for uncollectible accounts (01/01/X1) $ 30,000 Uncollectible accounts written off during 20X1 18,000 Uncollectible accounts recovered during 20X1 2,000 Accounts receivable on 12/31/X1 350,000 For 20X1, what would be Inge's uncollectible accounts expense?

Net write-offs of accounts receivable ($18,000 - $2,000) $16,000 Less decrease in "allowance" account ($30,000 - $25,000) (5,000) -------- Uncollectible accounts expense (i.e., credits to "allowance" account) $11,000 ========

The following information pertains to Spee Co.'s 20X1 sales: Cash Sales Gross $40,000 Returns and allowances 2,000 Credit Sales Gross 60,000 Discounts 3,000 On January 1, 20X1, customers owed Spee $20,000. On December 31, 20X1, customers owed Spee $15,000. Spee uses the direct write-off method for bad debts. No bad debts were recorded in 20X1. Under the cash basis of accounting, what amount of revenue should Spee report for 20X1?

One needs to convert from accrual to cash method income (revenue when collected in cash). Cash collected from cash sales ($40,000 - 2,000) = $ 38,000 Cash collected from credit sales: Net credit sales for 20X1 ($60,000 - 3,000) $57,000 January 1, 20X1, accounts receivable 20,000 ------- Subtotal $77,000 Less December 31, 20X1, accounts receivable 15,000 62,000 ------- -------- Cash basis revenue for 20X1 $100,000 ========

On December 31, 20X1 and 20X2, Carr Corp. had outstanding 4,000 shares of $100 par value 6% cumulative preferred stock and 20,000 shares of $10 par value common stock. On December 31, 20X1, dividends in arrears on the preferred stock were $12,000. Cash dividends declared in 20X2 totaled $44,000. Of the $44,000, what amounts were payable on each class of stock?

Preferred stock dividends for each year must be paid before common stock dividends may be paid. Cumulative preferred stock dividends in arrears must be caught up as well, prior to paying any dividends to common stock for the year. Dividends in arrears on preferred stock $12,000 Preferred stock dividend requirement for 20X1 (4,000 shares x $100 x 6%) 24,000 ------- Dividends allocable to preferred shares $36,000 ======= Dividends allocable to common shares ($44,000 - $36,000) $ 8,000 =======

On November 1, 20X1, Key Co. paid $3,600 to renew its only insurance policy for three years. On December 31, 20X1, Key's unadjusted trial balance showed a balance of $90 for prepaid insurance and $4,410 for insurance expense. What amounts should be reported for prepaid insurance and insurance expense in Key's December 31, 20X1, financial statements?

Prepaid insurance on November 1, 20X1 $3,600 Less November and December expense 2($3,600/36 months) 200 ------ Prepaid insurance on December 31, 20X1 $3,400 ====== Insurance expense Total payments Prepaid insurance on December 31, 20X1 = for insurance - balance = ($90 + $4,410) - $3,400 = $4,500 - $3,400 = $1,100

A company has an underfunded defined benefit pension plan. During the current year, the company uses the years-of-service method to amortize its prior service cost. What effect will the amortization of prior service cost have on the company's current-year financial statements?

Prior service cost is a benefit granted to current workers in respect of their prior service. It is recognized as an expense over the remaining service lives of the benefited workers. Until then, it is recognized as a part of the total projected benefit obligation, and debited to other comprehensive income (thus lowering other comprehensive income). As prior service cost is recognized as an expense, it is credited to (taken out of) other comprehensive income (thus increasing it).

Upon the death of an officer, Jung Co. received the proceeds of a life insurance policy held by Jung on the officer. The proceeds were not taxable. The policy's cash surrender value had been recorded on Jung's books at the time of payment. What amount of revenue should Jung report in its statements?

Proceeds received less cash surrender value

Quinn Co. reported a net deferred tax asset of $9,000 in its December 31, 20X1, balance sheet. For 20X2, Quinn reported pretax financial statement income of $300,000. Temporary differences of $100,000 resulted in taxable income of $200,000 for 20X2. At December 31, 20X2, Quinn had cumulative taxable differences of $70,000. Quinn's effective income tax rate is 30%. In its December 31, 20X2, income statement, what should Quinn report as deferred income tax expense?

Quinn Co. reported a net deferred tax asset of $9,000 in its December 31, 20X1, balance sheet. For 20X2, Quinn reported pretax financial statement income of $300,000. Temporary differences of $100,000 resulted in taxable income of $200,000 for 20X2. At December 31, 20X2, Quinn had cumulative taxable differences of $70,000. Quinn's effective income tax rate is 30%. In its December 31, 20X2, income statement, what should Quinn report as deferred income tax expense?

Luge Co., which began operations on January 2, 20X1, appropriately uses the installment sales method of accounting. The following information is available for 20X1: Installment accounts receivable, December 31, 20X1 (after reduction for 20X1 cash collections) $800,000 Deferred gross profit, December 31, 20X1 (before recognition of realized gross profit for 20X1) $560,000 Gross profit on sales 40% For the year ended December 31, 20X1, cash collections and realized gross profit on sales should be:

Remaining deferred gross profits must be based on uncollected cash from installment sales receivables. Upon collections in cash, gross profits are recognized at the gross profit percentage. The gross profit percentage of the uncollected installment receivables is the deferred gross profit. 20X1 Installment sales = 20X1 Deferred gross profit --------------------- Gross profit margin = $560,000 / .40 = $1,400,000 Cash collections in 20X1 = 20X1 Sales - 12/31/X1 Receivables = $1,400,000 - $800,000 = $600,000 20X1 Realized gross profit = Gross profit margin x Cash collections = 40% x 600,000 = $240,000

A company whose stock is trading at $10 per share has 1,000 shares of $1 par common stock outstanding when the board of directors declares a 30% common stock dividend. Which of the following adjustments should be made when recording the stock dividend?

Retained earnings is debited for $300.

Selected information from the accounts of Row Co. on December 31, 20X1, follows: Total income since incorporation $420,000 Total cash dividends paid 130,000 Total value of property dividends distributed 30,000 Excess of proceeds over cost of treasury stock sold, accounted for using cost method 110,000 In its December 31, 20X1, financial statements, what amount should Row report as retained earnings?

Retained earnings on December 31, 20X1, would be computed: Total income since incorporation $420,000 Less cash dividends $130,000 Property dividends 30,000 160,000 -------- -------- Retained earnings on December 31, 20X1 $260,000

During 20X1, Fleet Co.'s trademark was licensed to Hitch Corp. for royalties of 10% of net sales of the trademarked items. Returns were estimated to be 1% of gross sales. On signing the licensing agreement, Hitch paid Fleet $75,000 as an advance against future royalty earnings. Gross sales of the trademarked items during the year were $600,000. What amount should Fleet report as royalty income for 20X1?

Royalty income is Net sales × 10% = ($600,000 - $6,000) × 0.10 = $59,400.

Nest Co. issued 100,000 shares of common stock. Of these, 5,000 were held as treasury stock on December 31, 20X1. During 20X2, transactions involving Nest's common stock were as follows: May 3 - 1,000 shares of treasury stock were sold. August 6 - 10,000 shares of previously unissued stock were sold. November 18 - a 2-for-1 stock split took effect. Laws in Nest's state of incorporation protect treasury stock from dilution. On December 31, 20X2, how many shares on Nest's common stock were issued and outstanding?

Shares issued include: Original shares issued 100,000 Sale of shares on August 6 10,000 Additional shares from stock split 110,000 ------- Total 220,000 Shares outstanding: Change Outstanding -------- ----------- Original issue 100,000 100,000 Treasury shares held in 20X1 (5,000) 95,000 Sales of Treasury shares in 20X2 1,000 96,000 Sale of unissued shares 10,000 106,000 Stock split 106,000 212,000 The difference of 8,000 shares is the 4,000 treasury shares plus the additional 4,000 issued treasury shares in the stock split.

Brass Co. reported income before income tax expense of $60,000 for Year 2. Brass had no permanent or temporary timing differences for tax purposes. Brass has an effective tax rate of 30% and a $40,000 net operating loss carryforward from Year 1. What is the maximum income tax benefit that Brass can realize from the loss carryforward for Year 2?

Since all of the net operating loss can be used to offset income in Year 2, the maximum benefit is the amount of the net operating loss multiplied by the tax rate: $40,000 × 0.30 = $12,000

In September 20X1, West Corp. made a dividend distribution of one right for each of its 120,000 shares of outstanding common stock. Each right was exercisable for the purchase of 1/100 of a share of West's $50 variable rate preferred stock at an exercise price of $80 per share. On March 20, 20X5, none of the rights had been exercised, and West redeemed them by paying each stockholder $0.10 per right. As a result of this redemption, West's stockholders' equity was reduced by:

Summary journal entries for issuance and redemption of rights: Dr. Cr. September 20X1 Memo entry only March 20, 20X5, Retained Earnings 12,000 (120,000 x $.10) Cash 12,000 The result is a reduction of $12,000 in West's stockholders' equity.

The stockholders' equity section of Brown Co.'s December 31, 20X1, balance sheet consisted of the following: Common stock, $30 par, 10,000 shares authorized and outstanding $300,000 Additional paid-in capital 150,000 Retained earnings (deficit) (210,000) On January 2, 20X2, Brown put into effect a stockholder-approved quasi-reorganization by reducing the par value of the stock to $5 and eliminating the deficit against additional paid-in capital. Immediately after the quasi-reorganization, what amount should Brown report as additional paid-in capital?

Summary journal entries needed to effect quasi-reorganization: Dr. Cr. Common stock* 250,000 Additional paid-in capital 250,000 Additional paid-in capital 210,000 Retained earnings 210,000 * Common stock reduced by ($30 - $5) x 10,000 = $250,000 The resulting balance in additional paid-in capital is $150,000 + $250,000 - $210,000 = $190,000.

Asp Co. was organized on January 2, 20X1, with 30,000 authorized shares of $10 par common stock. During 20X1 the corporation had the following capital transactions: January 5 - issued 20,000 shares at $15 per share. July 14 - purchased 5,000 shares at $17 per share. December 27 - reissued the 5,000 shares held in treasury at $20 per share. Asp used the par value method to record the purchase and re-issuance of the treasury shares. In its December 31, 20X1, balance sheet, what amount should Asp report as additional paid-in capital in excess of par?

Summary journal entries: Dr. Cr. Jan. 5 Cash (20,000 x $15) 300,000 Additional paid-in capital (20,000 x ($15 - $10)) 100,000 Common stock (20,000 x $10) 200,000 July 14 Treasury stock (5,000 x $10) 50,000 Additional paid-in capital (5,000 x ($15 - $10)) 25,000* Retained earnings (5,000 x ($17 - $15)) 10,000 Cash (5,000 x $17) 85,000 Dec. 27 Cash (5,000 x $20) 100,000 Additional paid-in capital (5,000 x ($20 - $10)) 50,000 Treasury stock (5,000 x $10) 50,000 Balance of additional paid-in capital 12/31/X1 = $100,000 - $25,000 + $50,000 = $125,000 * (5,000 shares / 20,000 shares) x $100,000 = 25% x $100,000 = $25,000 OR * 5,000 shares x ($15 original issue price per share - $10 par value share) = $25,000

West, Inc., made the following expenditures relating to Product Y: Legal costs to file a patent on Product Y—$10,000. Production of the finished product would not have been undertaken without the patent. Special equipment to be used solely for development of Product Y—$60,000. The equipment has no other use and has an estimated useful life of four years. Labor and material costs incurred in producing a prototype model—$200,000. Cost of testing the prototype—$80,000. What is the total amount of costs that will be expensed when incurred?

The $10,000 patent cost will be capitalized as an intangible asset in accordance with the provisions of FASB ASC 350-30-25-1. The remaining costs are considered to be research and development costs, as defined in FASB ASC 730-10-55-1. Special developmental equipment and prototypes are addressed in FASB ASC 730-10-55-1. They are considered to be research and development costs which "shall be charged to expense when incurred." Thus the entire $340,000 ($60,000 + $200,000 + $80,000) would be expensed.

On January 1, Year 1, Alpha Co. signed an annual maintenance agreement with a software provider for $15,000 and the maintenance period begins on March 1, Year 1. Alpha also incurred $5,000 of costs on January 1, Year 1, related to software modification requests that will increase the functionality of the software. Alpha depreciates and amortizes its computer and software assets over five years using the straight-line method. What amount is the total expense that Alpha should recognize related to the maintenance agreement and the software modifications for the year ended December 31, Year 1?

The annual expenses would be the $15,000 maintenance contract multiplied by 10/12 of the year covered, or $15,000 × 10/12 = $12,500 from March to the end of the year. Also, expenses would cover 1/5 ($1,000) of the $5,000 from the other costs for one of the five years: $12,500 + $1,000 = $13,500 total.

On October 1 of the prior year, Fleur Retailers signed a 4-month, 16% note payable to finance the purchase of holiday merchandise. At that date, there was no direct method of pricing the merchandise, and the note's market rate of interest was 11%. Fleur recorded the purchase at the note's face amount. All of the merchandise was sold by December 1 of the prior year. Fleur's prior-year financial statements reported interest payable and interest expense on the note for three months at 16%. All amounts due on the note were paid February 1 of the current year. As a result of Fleur's accounting treatment of the note, interest, and merchandise, which of the following items was reported correctly?

The cost of the merchandise purchased (and sold by the end of the year) should have been based on the present value of the note used to pay for them, not the face amount. Since the note paid a higher rate of interest than what was required as a yield, the note would have a premium, a higher value than face. Thus, the note's present value was higher than its face amount, and the higher value should have been added to purchase cost and moved to cost of goods sold. The lower value that was used for purchase cost understated the cost of goods sold. If cost of goods sold was understated, then net income was wrong and retained earnings was not correct. Interest payable, however, is based on the face amount of the note and the stated payment rate, so it is correct.

Pine Corp.'s books showed pretax income of $800,000 for the current year ended December 31. In the computation of federal income taxes, the following data were considered: Gain on involuntary conversion (Pine has elected to replace the property within the statutory period using total proceeds.) $350,000 Depreciation deducted for tax purposes in excess of depreciation deducted for book purposes $50,000 Federal estimated tax payments $70,000 Enacted federal tax rates 30% What amount should Pine report as its current federal income tax liability on its December 31 balance sheet?

The current income tax liability is based on the taxable income for the year and the tax rate for the year, so one needs to compute taxable income first. The pretax income was $800,000 and part of this amount is a gain that qualifies for tax deferral, the reinvested proceeds from the involuntary conversion gain. Also, the amount of tax depreciation is used to compute taxable income. Taxable income is $400,000 ($800,000 less the gain of $350,000, and less the additional tax depreciation of $50,000). This amount multiplied by the tax rate of 30% gives us the tax due of $120,000 ($400,000 × 0.30). Because some of this amount due has been paid in estimates already, only the remaining $50,000 is a liability yet to pay ($120,000 - $70,000).

Leer Corp.'s pretax income in the current year was $100,000. The temporary differences between amounts reported in the financial statements and the tax return are as follows: Depreciation in the financial statements was $8,000 more than tax depreciation. The equity method of accounting resulted in financial statement income of $35,000. A $25,000 dividend was received during the year, which is eligible for the 80% dividends-received deduction. Leer's effective income tax rate was 30%. In its current year income statement, Leer should report a current provision for income taxes of:

The current provision for income taxes is simply the taxable income for the year multiplied by the tax rate for the year. Thus, we need to find the taxable income for the year. Start with the pretax income of $100,000, and add $8,000 to it because tax depreciation expense was less than book depreciation. The $35,000 equity method income for financial accounting needs to be subtracted since it is not the taxed amount: $100,000 + $8,000 - $35,000 = $73,000 The dividends that are taxable are subject to a dividends-received deduction of 80%. Thus, only add in $5,000 of the dividends ($25,000 × 0.20 (1 − 0.80)), because the dividends, though taxable in part, are not financial accounting income when applying the equity method: $73,000 + $5,000 = $78,000 Thus, taxable income is $78,000 ($100,000 + $8,000 - $35,000 + $5,000) and the current income tax due is $23,400: $78,000 × 0.30 = $23,400

For a defined benefit pension plan, the discount rate used to calculate the projected benefit obligation is determined by:

The discount rate for the projected benefit obligation is generally based on long-term debt interest rates. It has no direct relationship to the actual or the expected rate of return on plan assets.

Which of the following statements is correct regarding valuation allowances in accounting for income taxes?

The effect of a change in the opening balance of a valuation allowance that results from a change of circumstances ordinarily is included in income from operations

Frame Construction Company's contract requires the construction of a bridge in 3 years. The expected total cost of the bridge is $2,000,000, and Frame will receive $2,500,000 for the project. The actual costs incurred to complete the project were $500,000, $900,000, and $600,000, respectively, during each of the 3 years. Progress payments received by Frame were $600,000, $1,200,000, and $700,000, respectively. Assuming that the percentage-of-completion method is used, what amount of gross profit would Frame report during the last year of the project?

The first step is to figure the total profit on the contract, as follows: Total revenue was $2,500,000. Total actual costs are known (since the project has been completed) to be $2,000,000 (made up of $500,000 + $900,000 + $600,000 from the 3 years of work). This gives us a profit of $500,000 on the contract ($2,500,000 - $2,000,000). At the beginning of the third year, Frame had expended a total cost of $1,400,000 ($500,000 from the first year, and $900,000 more from the second year combined). At the start of Year 3, Frame was thus 70% complete (based on total cost expended so far, $1,400,000, divided by total cost estimated to finish, $2,000,000). Frame would have already recognized 70% of the total contract profit so far ($350,000, or 0.7 × $500,000 total profit). Thus, Frame has only $150,000 profit remaining to be recognized in Year 3 (Total profit of $500,000 - Profit already recognized of $350,000). Since the total expected cost was the total actual cost, in this particular case the percentage completed in Year 3 times the total contract profit will also give the correct answer: $600,000 ÷ $2,000,000 = 0.3 0.3 × $500,000 = $150,000

Fern Co. has net income, before taxes, of $200,000, including $20,000 interest revenue from municipal bonds and $10,000 paid for officers' life insurance premiums where the company is the beneficiary. The tax rate for the current year is 30%. What is Fern's effective tax rate?

The first step is to know the taxable income. Net income for accounting purposes before tax would be the starting point. Adjust this number up or down for nontaxable or nondeductible items. The municipal interest income would be nontaxable, so this would be subtracted from accounting income and the insurance premiums would be nondeductible (since they relate to nontaxed income) and would need to be added. Thus, net income before taxes of $200,000 minus the municipal bond interest of $20,000, plus the insurance premiums $10,000 equals taxable income of $190,000: $200,000 - $20,000 + $10,000 = $190,000 Taxable income times the tax rate equals tax due of $57,000: $190,000 × 0.30 = $57,000 The effective tax rate would be the total tax due divided by the total income earned: $57,000 ÷ $200,000 = 0.285 (28.5%)

An entity purchased new machinery from a supplier before the entity's year-end. The entity paid freight charges for the purchased machinery. The entity took out a loan from a bank to finance the purchase. Under IFRS, what is the proper accounting treatment for the freight and interest costs related to the machinery purchase?

The freight cost should be capitalized as part of property, plant, and equipment, and the interest cost should be immediately expensed.

Asp Co. appropriately uses the installment method of revenue recognition to account for its credit sales. The following information was abstracted from Asp's December 31, Year 2, financial statements: Year 2 Year 1 ---------- ---------- Sales $1,500,000 $1,000,000 Accounts receivable: Year 2 sales 900,000 Year 1 sales 540,000 600,000 Deferred gross profit: Year 2 sales 252,000 Year 1 sales 108,000 120,000 What was Asp's gross profit percentage for Year 2 sales?

The gross profit percentage is Gross profit ÷ Sales: $252,000 ÷ $900,000 = 0.28 (28%)

On January 2, Year 1, Ross Co. purchased a machine for $70,000. This machine has a 5-year useful life, a residual value of $10,000, and is depreciated using the straight-line method for financial statement purposes. For tax purposes, depreciation expense was $25,000 for Year 1 and $20,000 for Year 2. Ross's Year 2 income, before income taxes and depreciation expense, was $100,000 and its tax rate was 30%. If Ross had made no estimated tax payments during Year 2, what amount of current income tax liability would Ross report in its December 31, Year 2, balance sheet?

The income tax due and yet to be paid for the year (payable) is $24,000. The income before income taxes and depreciation is $100,000 and the tax depreciation is taken from that to compute taxable income. Taxable income is $80,000 ($100,000 - $20,000) and the tax rate is 30%, so the current income tax due is $24,000 ($80,000 × 0.30). No part of the tax has been paid (no estimates for the year, yet), so it is all still payable.

In its Year 1 income statement, Tow, Inc., reported proceeds from an officer's life insurance policy of $90,000 and depreciation of $250,000. Tow was the owner and beneficiary of the life insurance on its officer. Tow deducted depreciation of $370,000 in its Year 1 income tax return when the tax rate was 30%. Data related to the reversal of the excess tax deduction for depreciation follow: Reversal of Enacted Year Excess Tax Deduction Tax Rates 2 $50,000 35% 3 40,000 35% 4 20,000 25% 5 10,000 25% There are no other temporary differences. Tow expects to report profits (rather than losses) for tax purposes for all future years. In its December 31, Year 1, balance sheet, what amount should Tow report as a deferred income tax liability?

The life insurance proceeds will not be taxed ever, but the depreciation is taxed at a different times than when it is reported on the financial records. The tax depreciation is taken first, so later years will have fewer deductions and more taxes due. The amounts due in the future are based on the times of reversal and the rates in effect at the time. For Year 2, $50,000 × 0.35 = $17,500. For Year 3, $40,000 × 0.35 = $14,000. For Year 4, $20,000 × 0.25 = $5,000. For Year 5, $10,000 × 0.25 = $2,500. Adding these amounts results in the total deferred income tax liability for Year 1: $17,500 + $14,000 + $5,000 + $2,500 = $39,000

On January 2, 20X2, Lake Mining Co.'s board of directors declared a cash dividend of $400,000 to stockholders of record on January 18, 20X2, payable on February 10, 20X2. The dividend is permissible under law in Lake's state of incorporation. Selected data from Lake's December 31, 20X1, balance sheet are as follows: Accumulated depletion $100,000 Capital stock 500,000 Additional paid-in capital 150,000 Retained earnings 300,000 The $400,000 dividend includes a liquidating dividend of:

The liquidating dividend is that portion of the cash dividend that exceeds the balance in retained earnings because other equity accounts must be debited. Thus, for Lake: Total amount of January 2, 20X2, cash dividend $400,000 Less: Retained earnings balance 300,000 -------- Liquidating dividend $100,000 ======== Liquidating dividends are a return of the investment rather than a return on the investment.

On January 2, 20X1, Lem Corp. bought machinery under a contract that required a down payment of $10,000, plus 24 monthly payments of $5,000 each, for total cash payments of $130,000. The cash equivalent price of the machinery was $110,000. The machinery has an estimated useful life of 10 years and estimated salvage value of $5,000. Lem uses straight-line depreciation. In its 20X1 income statement, what amount should Lem report as depreciation for the machinery?

The machinery is recorded at the cash equivalent price of the machinery, $110,000. Straight-line depreciation for 20X1 = $(110,000 - $5,000) / 10 years = $ 105,000 / 10 y

Lion Co.'s income statement for its first year of operations shows pretax income of $6,000,000. In addition, the following differences existed between Lion's tax return and records: Tax Accounting Return Records -------- ---------- Uncollectible accounts expense $220,000 $250,000 Depreciation expense 860,000 570,000 Tax-exempt interest revenue - 50,000 Lion's current-year tax rate is 30% and the enacted rate for future years is 40%. What amount should Lion report as deferred tax expense in its income statement for the year?

The permanent difference from tax-exempt interest does not produce deferred taxes. Uncollectible accounts ($220,000 - $250,000) $(30,000) Depreciation expense ($860,000 - $570,000) 290,000 --------- Net temporary differences $260,000 Tax rate x .40 --------- Deferred tax expense $104,000

Taft Corp. uses the equity method to account for its 25% investment in Flame, Inc. During 20X1, Taft received dividends of $30,000 from Flame and recorded $180,000 as its equity in the earnings of Flame. Additional information follows: All the undistributed earnings of Flame will be distributed as dividends in future periods. The dividends received from Flame are eligible for the 80% dividends received deduction. There are no other temporary differences. Enacted income tax rates are 30% for 20X1 and thereafter. In its December 31, 20X1, balance sheet, what amount should Taft report for deferred income tax liability?

The recognized but as yet unreceived income will generate a future tax consequence, a future tax liability. However, dividends received by a corporation are eligible for a dividends-received deduction, and thus only a smaller taxable amount (multiplied by the tax rate) will eventually be paid. Taft's recorded equity in Flame earnings $180,000 Less dividends received in 20X1 30,000 -------- Temporary difference before dividend deduction 150,000 Less dividends received deduction (80% x $150,000) 120,000 -------- Net amount of temporary difference $ 30,000 Times tax rate x 30% -------- Deferred income tax liability, noncurrent $ 9,000 ========

On January 2, 20X1, Lava, Inc., purchased a patent for a new consumer product for $90,000. At the time of purchase, the patent was valid for 15 years; however, the patent's useful life was estimated to be only 10 years due to the competitive nature of the product. On December 31, 20X4, the product was permanently withdrawn from sale under governmental order because of a potential health hazard in the product. What amount should Lava charge against income during 20X4, assuming that amortization is recorded at the end of each year?

The remaining unamortized cost of any asset that is totally impaired is a loss. Cost of patent $90,000 Patent amortization for 20X1 through 20X3 (($90,000 / 10 years) x 3 years) 27,000 ------- Carrying value of patent on 01/01/X4 (amount written off in 20X4) $63,000 =======

Wren Corp.'s trademark was licensed to Mont Co. for royalties of 15% of sales of the trademarked items. Royalties are payable semiannually on March 15 for sales in July through December of the prior year, and on September 15 for sales in January through June of the same year. Wren received the following royalties from Mont: March 15 September 15 Year 1 $10,000 $15,000 Year 2 12,000 17,000 Mont estimated that sales of the trademarked items would total $60,000 for July through December, Year 2. In Wren's Year 2 income statement, the royalty revenue should be:

The royalty revenue should be based on the trademark sales during Year 2 (15% of them). The amount received on September 15 of Year 2, $17,000, was for the sales for the first 6 months of Year 2, and the estimated sales for the last 6 months of Year 2 were $60,000 in total. The royalty revenue for Year 2 was the $17,000 plus 15% of the $60,000 for a total of $26,000 ($17,000 + (0.15 × $60,000)).

During January of the previous year, Doe Corp. agreed to sell the assets and product line of its Hart division. The sale on January 15 of the current year resulted in a gain on disposal of $900,000. Not considering any impairment losses, Hart's operating losses were $600,000 for the previous year and $50,000 for the current-year period January 1 through January 15. Disregarding income taxes, what amount of net gain (loss) should be reported in Doe's comparative current and previous years' income statements?

The sale of a division would be a discontinued operation since its disposition represents a strategic shift. The discontinued operation would be recorded in the year the sale occurred. Previous Current Net loss from continuing operations $(600,000) $(50,000) Gain on sale of discontinued operations 900,000 Net income $(600,000) $850,000

On September 1, Year 1, Howe Corp., offered special termination benefits to employees who had reached the early retirement age specified in the company's pension plan. The termination benefits consisted of lump-sum and periodic future payments. Additionally, the employees accepting the company offer receive the usual early retirement pension benefits. The offer expired on November 30, Year 1. Actual or reasonably estimated amounts at December 31, Year 1, relating to the employees accepting the offer are as follows: Lump-sum payments totaling $475,000 were made on January 1, Year 2. Periodic payments of $60,000 annually for three years will begin January 1, Year 3. The present value at December 31, Year 1, of these payments was $155,000. Reduction of accrued pension costs at December 31, Year 1, for the terminating employees was $45,000. In its December 31, Year 1, balance sheet, Howe should report a total liability of special termination benefits of:

The total liability that needs to be recognized is for the lump sum and the present value of the periodic payments ($475,000 + $155,000 = $630,000). It is not to be offset against the reduction in accrued pension costs.

Jan Corp. amended its defined benefit pension plan, granting a total credit of $100,000 to four employees for services rendered prior to the plan's adoption. The employees, A, B, C, and D, are expected to retire from the company as follows: "A" will retire after three years. "B" and "C" will retire after five years. "D" will retire after seven years. What is the amount of prior service cost amortization in the first year?

The total number of Future Service Years is 20; the total number in the first year is 4. Year 1 amortized prior service cost is 4/20 × $100,000 = $20,000.

On January 2, 20X1, Loch Co. established a noncontributory defined-benefit pension plan covering all employees and contributed $400,000 to the plan. At December 31, 20X1, Loch determined that the 20X1 service and interest costs on the plan were $720,000. The expected and the actual rate of return on plan assets for 20X1 was 10%. There are no other components of Loch's pension expense. What amount should Loch report as accrued pension cost in its December 31, 20X1, balance sheet?

The unfunded part of the total pension cost is the total in accrued pension cost. The service and interest costs for the year add to it, and the contributions and plan return lower it. 20X1 service and interest cost $720,000 Less: Contribution to plan $400,000 Interest on contribution (10% of $400,000) 40,000 440,000 -------- -------- Accrued pension cost $280,000 ========

On January 1, Year 1, a company issued its employees 10,000 shares of restricted stock. On January 1, Year 2, the company issued to its employees an additional 20,000 shares of restricted stock. Additional information about the company's stock is as follows: Fair Value of Stock Date (per share) ------------------- ------------------- January 1, Year 1 $20 December 31, Year 1 22 January 1, Year 2 25 December 31, Year 2 30 The shares vest at the end of a 4-year period. There are no forfeitures. What amount should be recorded as compensation expense for the 12-month period ended December 31, Year 2?

The value of the restricted stock is earned equally over the 4-year vesting period, and the compensation expenses must be divided equally into those years. The total expense is the value of the shares multiplied by the amounts: 10,000 × $20 = $200,000, and also 20,000 × $25 for an additional $500,000, for a total of $700,000 compensation expense, earned 1/4th each year: $700,000 × 0.25 = $175,000.

At the beginning of Year 1, a company hired an executive whose contract included the promise of payment of $100,000 in each of Years 6, 7, and 8, if the executive is employed at the end of Year 5. How should the compensation expense associated with this contract be recorded?

This is a deferred compensation arrangement, and the cost of the future payments must be accrued as earned. The payments of $100,000 paid each year over three years total $300,000 and, if they are equally earned over Years 1 through 5, then they are an expense at the rate of $60,000 a year ($300,000 ÷ 5).

A shoe retailer allows customers to return shoes within 90 days of purchase. The company estimates that 5% of sales will be returned within the 90-day period. During the month, the company has sales of $200,000 and returns of sales made in prior months of $5,000. What amount should the company record as net sales revenue for new sales made during the month?

This month's estimated returns are all that matter. Thus, $200,000 × (1 - 0.05) is the month's net sales: $200,000 × 0.95 = $190,000

As a result of differences between depreciation for financial reporting purposes and tax purposes, the financial reporting basis of Noor Co.'s sole depreciable asset, acquired in 20X1, exceeded its tax basis by $250,000 at December 31, 20X1. The difference will reverse in future years. The enacted tax rate is 30% for 20X1 and 40% for future years. Noor has no other temporary differences. In its December 31, 20X1, balance sheet, how should Noor report the deferred tax effect of this difference?

This temporary difference will result in additional taxes being paid in future years so the related tax effect will be a liability. Amount of liability = Temporary difference x Future enacted tax rate = $250,000 x 0.40 = $100,000

Bounty Co. provides postretirement health care benefits to employees who have completed at least 10 years service and are age 55 years or older when retiring. Employees retiring from Bounty have a median age of 62, and no one has worked beyond age 65. Fletcher is hired at 48 years old. The attribution period for accruing Bounty's expected postretirement health care benefit obligation to Fletcher is during the period when Fletcher is age:

Thus, Bounty Co.'s attribution period for Fletcher covers the period when Fletcher is age 48 (date hired) to 58 (when the 10-years-of-service requirement is met for full benefit eligibility).

The following information pertains to Seda Co.'s pension plan for the current year: Actuarial estimate of projected benefit obligation at 1/1 $72,000 Assumed discount rate 10% Service costs for the year $18,000 Pension benefits paid during the year $15,000 If no change in actuarial estimates occurred during the year, Seda's projected benefit obligation at December 31 of the current year was:

To compute projected benefit obligation at the end of the year, start with the beginning-of-year balance, add service cost, add interest cost, and subtract out benefits paid out during the year. Interest cost is computed by multiplying the discount rate times the beginning year balance, so interest cost is: $72,000 × 0.10 = $7,200 Thus, the end-of-year balance is: $72,000 + $7,200 + $18,000 - $15,000 = $82,200

The following information pertains to Lee Corp.'s defined benefit pension plan for the current year: Service cost $160,000 Actual and expected gain on plan assets 35,000 Unexpected loss on plan assets related to a disposal of a subsidiary 40,000 Amortization of prior service cost 5,000 Annual interest on pension obligation 50,000 What amount should Lee report as pension expense in its current year-end income statement?

To compute the pension expense, add the service cost, subtract the expected rate of return on plan assets, add the amortization of prior service cost, and add the interest cost on the pension obligation: $160,000 - $35,000 + $5,000 + $50,000 = $180,000

For the year ended December 31, 20X1, Tyre Co. reported pretax financial statement income of $750,000. Its taxable income was $650,000. The difference is due to accelerated depreciation for income tax purposes. Tyre's effective income tax rate is 30%, and Tyre made estimated tax payments during 20X1 of $90,000. What amount should Tyre report as current income tax expense for 20X1?

Tyre's current income tax expense is simply taxable income multiplied by the effective income tax rate. (Note: Tyre also has a noncurrent deferred income tax liability in the amount of $30,000 ($100,000 timing difference multiplied by the 30% tax rate).) Current income tax expense = Taxable income x Tax rate = $650,000 x 0.30 = $195,000

As of December 31 of the current year, the accumulated postretirement benefit obligation and plan assets of a defined benefit postretirement plan sponsored by Crouse, Inc., were: Accumulated postretirement benefit obligation $500,000 Plan assets at fair value 425,000 Transition obligation $ 75,000 ======== Crouse elected to apply GAAP provisions for employers' accounting for postretirement benefits other than pensions, in its financial statements for the current year ended December 31 and recognize the transition amount on a delayed basis as a component of net periodic postretirement benefit cost. The average remaining service period of active plan participants expected to receive benefits was estimated to be 10 years at the date of transition. Some participants' estimated service periods are 25 years. To minimize an accrual for postretirement benefit cost, what amount of transition obligation should Crouse amortize?

Under the circumstances given, the transition obligation can be amortized over a 20-year period. Since the average remaining service life of active participants is under 20 years, the longer 20-year period is available, and thus the annual amortization is $3,750 ($75,000 ÷ 20 years). (FASB ASC 715-60-35-39)

Information pertaining to dividends from Wray Corp.'s common stock investments for the year ending December 31, 20X1, follows: On September 8, 20X1, Wray received a $50,000 cash dividend from Seco, Inc., in which Wray owns a 30% interest. A majority of Wray's directors are also directors of Seco. On October 15, 20X1, Wray received a $6,000 liquidating dividend from King Co. Wray owns a 5% interest in King Co. Wray owns a 2% interest in Bow Corp., which declared a $200,000 cash dividend on November 27, 20X1, to stockholders of record on December 15, 20X1, payable on January 5, 20X2. What amount should Wray report as dividend income in its income statement for the year ending December 31, 20X1?

Under the cost method, dividends are recorded as income (revenue) when the dividend is declared. Dividend income for 20X1 = Dividends from Bow Corp. = 0.02 x $200,000 = $4,000 The investment in Seco, Inc., would be accounted for using the equity method so the dividends would be treated as "recovery of investment." Under the equity method, a proportionate share of income is recognized "as earned." The liquidating dividend from King Co. is not income, but rather a return of investment to owners.

Entor Co. sold equipment to Pane Co. for $50,000. The equipment had a net book amount of $30,000. The collections were $20,000 in the first year, $15,000 in the next year, and $15,000 in the last year. What is the amount of gross profit for the third year if Entor used the installment sales accounting method for the transaction?

Under the installment sales method, the gross profit on sales is deferred and recognized as cash is actually collected. The gross profit percentage is the realized gain divided by the contract price. This gross profit percentage is multiplied by any cash received to determine the gain to be included in net income. Cash received in the third year $15,000 Gross profit percentage $20,000/$50,000 = 0.40 ------- Gross profit in Year 3 $ 6,000

When should an anticipated loss on a long-term contract be recognized under the percentage-of-completion method and the completed-contract method, respectively?

Under the percentage-of-completion method, gains are recognized on profitable long-term contracts as the work is progressing, and under the completed-contract method, all gains are deferred and recognized when the contract is complete. However, both methods are alike in recognizing estimated ultimate losses on a losing contract immediately.

On January 1 of the current year, Lundy Corp. purchased 40% of the voting common stock of Glen, Inc., and appropriately accounts for its investment by the equity method. During the year, Glen reported earnings of $225,000 and paid dividends of $75,000. Lundy assumes that all of Glen's undistributed earnings will be distributed as dividends in future periods when the enacted tax rate will be 30%. Ignore the dividends-received deduction. Lundy's current enacted income tax rate is 25%. Lundy uses the liability method to account for temporary differences and expects to have taxable income in all future periods. The increase in Lundy's deferred income tax liability for this temporary difference is:

When applying the equity method to an investment for financial accounting purposes, the income earned by the company partially owned is recognized by the owning investing company on its own books. Lundy has financial accounting income of $90,000 ($225,000 × 0.40) and this income is not recognized for tax purposes until received in dividends later on. Of course, Lundy did receive some dividends already, $30,000 ($75,000 × 0.40). Thus, $60,000 of deferred income for tax purposes will generate a future tax due, a deferred tax liability now of $18,000 ($60,000 × the future tax rate of 0.30). All deferred tax liabilities and deferred tax assets are classified on the balance sheet as noncurrent.

When would a company use the installment sales method of revenue recognition?

When installment sales are material, and there is no reasonable basis for estimating collectibility

At the beginning of year 1, a company amends its defined benefit pension plan for an additional $500,000 in prior service cost. The amendment covers employees with a 10-year average remaining service life. At the end of year 1, what is the net entry to Accumulated Other Comprehensive Income (AOCI), ignoring income tax effects?

When plan amendments are made, additional benefits are sometimes applied retroactively to employees for service rendered in prior years. This increase in the benefits represents a cost to the employer that GAAP requires to be recognized as a component of pension expense over the remaining service years of the affected employees. The unrecognized prior service cost must be recognized as a component of Other Comprehensive Income. The amortization of the unrecognized prior service cost is recognized as an increase in pension expense and as a reduction of the unrecognized amount remaining in Accumulated Other Comprehensive Income. At the end of year 1, 1/10 or $50,000 amortization would reduce the $500,000 unrecognized prior service cost, so the net entry to AOCI would be a debit of $450,000.

Parker Co. amended its pension plan on January 2 of the current year. It also granted $600,000 of unrecognized prior service costs to its employees. The employees are all active and expect to provide 2,000 service years in the future, with 350 service years this year. What is Parker's unrecognized prior service cost amortization for the year?

When plan amendments are made, additional benefits are sometimes applied retroactively to employees for service rendered in prior years. This increase in the benefits to be paid to employees represents a cost to the employer. GAAP requires that this cost be recognized as a component of pension expense over the remaining service years of the affected employees. FASB ASC 715-30-25-4 requires that the unrecognized prior service cost be recognized as a component of other comprehensive income. The amortization of the unrecognized prior service cost is recognized as an increase in pension expense and as a reduction of the unrecognized amount remaining in accumulated other comprehensive income. Prior service costs $600,000 Expected service years / 2,000 -------- Cost per service year $ 300 Service years completed this year x 350 -------- Unrecognized prior service cost amortization $105,000

On January 1, 20X1, Sip Co. signed a 5-year contract enabling it to use a patented manufacturing process beginning in 20X1. A royalty is payable for each product produced, subject to a minimum annual fee. Any royalties in excess of the minimum will be paid annually. On the contract date, Sip prepaid a sum equal to two years' minimum annual fees. In 20X1, only minimum fees were incurred. The royalty prepayment should be reported in Sip's December 31, 20X1, financial statements as:

a current asset and an expense.

Cash collection is a critical event for income recognition in:

both the cost recovery method and the installment method.

In accounting for a long-term construction contract using the percentage-of-completion method, the progress billings on contracts account is a:

contra current asset account. The current asset account maintaining an inventory value for the costs and profits recognized so far on the contract has a contra account of progress billings, lowering its carrying value. If the billings exceed the construction in process, then a current liability can exist instead.

The provisions of FASB ASC 718-10-25-2, "Recognition Principle for Share-Based Payment Transactions," apply to all of the following transactions except those related to:

employee stock ownership plan instruments.

Senlo Co. uses a 1-year operating cycle and recognizes profits for financial statement and tax purposes during its two years of operation. Depreciation for tax purposes exceeded depreciation for financial statement purposes each year. These temporary differences are expected to reverse in Years 3, 4, and 5. At the end of Year 2, the deferred tax liability shown as a noncurrent liability is based on the:

enacted tax rates for Years 3, 4, and 5

When the effective interest method of amortization is used for bonds issued at a premium, the amount of interest payable for an interest period is calculated by multiplying the:

face value of the bonds at the beginning of the period by the contractual interest rate.

For a defined benefit postretirement plan, an employer recognizes a transition obligation or transition asset, determined as the measurement date for the beginning of the fiscal year in which GAAP employers' accounting for postretirement benefits other than pensions is applied. The transition obligation or transition asset may be recognized:

immediately in net income of the period of change as the effect of a change in accounting principle and on a delayed basis as a component of net periodic postretirement benefit cost.

According to the installment method of accounting, gross profit on an installment sale is recognized in income:

in proportion to the cash collection.

Baker Co. uses the calendar year as its accounting year. During 20X1, Congress enacted new tax legislation that changed the tax rate for 20X2 from 30% to 40%. The tax rate for 20X3 and following years remained at 30%. Baker has only one type of temporary difference or carryforward—a taxable temporary difference. Accordingly, Baker had a deferred tax liability at the beginning of 20X1 and will have a deferred tax liability at the end of 20X2. With regard to the change in tax rates, Baker should:

include the effect of the change on the January 1, 20X1, deferred tax liability in income from continuing operations of 20X1.

Interest cost included in the net pension cost recognized by an employer sponsoring a defined benefit pension plan represents the:

increase in the projected benefit obligation due to the passage of time.

An overfunded single-employer defined benefit postretirement plan should be recognized in a classified statement of financial position as a:

noncurrent asset.

Because Jab Co. uses different methods to depreciate equipment for financial statement and income tax purposes, Jab has temporary differences that will reverse during the next year and add to taxable income. Deferred income taxes that are based on these temporary differences should be classified in Jab's balance sheet as a:

noncurrent liability.

Hut Co. has temporary taxable differences that will reverse during the next year and add to taxable income. These differences relate to noncurrent assets. Deferred income taxes based on these temporary differences should be classified in Hut's balance sheet as a:

noncurrent liability.

Drew Co. produces expensive equipment for sale on installment contracts. When there is doubt about eventual collectibility, the income recognition method least likely to overstate income is:

the cost recovery method.

On October 1 of the prior year, Fleur Retailers signed a 4-month, 16% note payable to finance the purchase of holiday merchandise. At that date, there was no direct method of pricing the merchandise, and the note's market rate of interest was 11%. Fleur recorded the purchase at the note's face amount. All of the merchandise was sold by December 1 of the prior year. Fleur's prior-year financial statements reported interest payable and interest expense on the note for three months at 16%. All amounts due on the note were paid February 1 of the current year. Fleur's prior-year cost of goods sold for the holiday merchandise was:

understated by the difference between the note's face amount and the note's October 1 present value.

Band Co. uses the equity method to account for its investment in Guard, Inc., common stock. How should Band record a 2% stock dividend received from Guard?

As a memorandum entry reducing the unit cost of all Guard stock owned

Generally, which inventory costing method approximates most closely the current cost for each of the following?

Cost of goods sold: LIFO; Ending inventory: FIFO

On November 2, 20X1, Finsbury, Inc., issued warrants to its stockholders giving them the right to purchase additional $20 par value common shares at a price of $30. The stockholders exercised all warrants on March 1, 20X2. The shares had market prices of $33, $35, and $40 on November 2, 20X1, December 31, 20X1, and March 1, 20X2, respectively. What were the effects of the warrants on Finsbury's additional paid-in-capital and net income?

Additional paid-in capital increased in 20X2; no effect on net income.

On December 31, 20X1, Key Co. received two $10,000 non-interest-bearing notes from customers in exchange for services rendered. The note from Alpha Co., which is due in nine months, was made under customary trade terms, but the note from Omega Co., which is due in two years, was not. The market interest rate for both notes at the date of issuance is 8%. The present value of $1 due in nine months at 8% is 0.944. The present value of $1 due in two years at 8% is 0.857. At what amounts should these two notes receivable be reported in Key's December 31, 20X1, balance sheet?

Alpha $10,000, Omega $8,570 Current receivables acquired as a result of customary trade terms are normally reported at their face value. Long-term receivables are reported at their present value: $10,000 × 0.857 = 8,570.

A company issued a bond with a stated rate of interest that is less than the effective interest rate on the date of issuance. The bond was issued on one of the interest payment dates. What should the company report on the first interest payment date?

An interest expense that is greater than the cash payment made to bondholders

Last year, Katt Co. reduced the carrying amount of its long-lived assets used in operations from $120,000 to $100,000, in connection with its annual impairment review. During the current year, Katt determined that the fair value of the same assets had increased to $130,000. What amount should Katt record as restoration of previously recognized impairment loss in the current year's financial statements?

$0 After an impairment loss is recognized, the reduced carrying amount of the asset should be treated as the new cost and the restoration of the impairment is not recognized. IFRS will allow the restoration of an impairment loss, but U.S. GAAP will not allow the restoration.

The following are held by Smite Co.: Cash in checking account $20,000 Cash in bond sinking fund account 30,000 Postdated check from customer dated 1 month from balance sheet date 250 Petty cash 200 Commercial paper (matures in 2 months) 7,000 Certificate of deposit (matures in 6 months) 5,000 What amount should be reported as cash and cash equivalents on Smite's balance sheet?

$27,200 The sinking fund would be restricted cash, and the postdated check is still a receivable.

Wall Co. sells a product under a 2-year warranty. The estimated cost of warranty repairs is 2% of net sales. During Wall's first two years in business, it made the following sales and incurred the following warranty repair costs: Year 1 ------ Total sales $250,000 Total repair costs incurred 4,500 Year 2 ------ Total sales $300,000 Total repair costs incurred 5,000 What amount should Wall report as warranty expense for Year 2?

In order to properly match warranty expense with the revenue generating the expenses, 2% of Year 2's sales (0.02 × $300,000 = $6,000) should be expensed.

On October 1, Year 1, Gold Co. borrowed $900,000 to be repaid in three equal, annual installments. The note payable bears interest at 5% annually. Gold paid the first installment of $300,000 plus interest on September 30, Year 2. What amount should Gold report as a current liability on December 31, Year 2?

Current liabilities will include any installments of principal due within the next 12 months, plus the interest expense accrued (but not paid) until the end of the year. The second installment of $300,000 is due on September 30, Year 3, so it is a current liability as of December 31, Year 2. The first installment was paid on September 30, Year 2, so only 2/3rds of the $900,000 is still outstanding as principal on December 31, Year 2; only $600,000 is left. The last payment on September 30, Year 2, included all interest to date; thus, we only need to accrue interest from September 30, Year 2, until December 31, Year 2, which is a total of $7,500 based on principal of $600,000 × 0.05 × 3/12 (last 3 months of the year). So, the total current liability as of December 31, Year 2, is $300,000 + $7,500, for a total of $307,500.

When the fair value of an investment in debt securities exceeds its amortized cost, how should each of the following debt securities be reported at the end of the year?

Debt securities classified as held-to-maturity should be reported as amortized cost, and debt securities classified as available-for-sale should be reported as fair value.

A company decided to change its inventory valuation method from FIFO to LIFO in a period of rising prices. What was the result of the change on ending inventory and net income in the year of the change?

Decrease in both ending inventory and net income

Grid Corp. acquired some of its own common shares at a price greater than both their par value and original issue price but less than their book value. Grid uses the cost method of accounting for treasury stock. What is the impact of this acquisition on total stockholders' equity and the book value per common share?

Decrease in total stockholders' equity and increase in book value per share

On July 1, 2015, Eagle Corp. issued 600 of its 10%, $1,000 bonds at 99 plus accrued interest. The bonds are dated April 1, 2015, and mature on April 1, 2021. Interest is payable semiannually on April 1 and October 1. What amount did Eagle receive from the bond issuance?

Eagle received $609,000: Sales price = 600 x $1,000 x 0.99 = $594,000 Accrued interest = 600 x $1,000 x 0.10 x (3/12) = 15,000 -------- Total amount received $609,000 ========

Beach Co. determined that the decline in the fair market value (FMV) of an investment was below the amortized cost and permanent in nature. The investment was classified as available-for-sale on Beach's books. The controller would properly record the decrease in FMV by including it in which of the following?

Earnings section of the income statement and writing down the cost basis to FMV

Could current cost financial statements report holding gains for goods sold during the period and holding gains on inventory at the end of the period?

Goods sold: Yes; Inventory: Yes

Which of the following statements is correct when a company applying the lower of cost or market method reports its inventory (accounted for under the LIFO method) at replacement cost?

II only

Rabb Co. records its purchases at gross amounts but wishes to change to recording purchases net of purchase discounts. Discounts available on purchases recorded from last October 1 to this September 30 totaled $2,000. Of this amount, $200 is still available in the accounts payable balance. The balances in Rabb's accounts as of and for the current year ended September 30 before conversion are: Purchases $100,000 Purchase discounts taken 800 Accounts payable 30,000 What is Rabb's current-year accounts payable balance as of September 30 after the conversion?

The difference between gross and net reporting is that at gross reporting, the discounts are not recognized in the carrying values of the accounts until payment is made. Thus, the accounts in question will be carried at their full gross amounts due (not less the discount available). The account that will be affected by the change is the accounts payable account that keeps track of the payments still due, at their full gross amount due of $30,000. The purchases already paid for have been adjusted for any available discount and do not require adjustment now. Any expired discounts are also no longer available and any purchases they relate to should stay at gross amounts due. The unexpired discounts that are still available to take, the $200, should be adjusted into the carrying value of accounts payable now still outstanding, and that is the only adjustment to make. Thus, what needs to be done is to restate accounts payable down by the $200 unexpired discounts, from $30,000 to $29,800.

A company issues bonds at 98, with a maturity value of $50,000. The entry the company uses to record the original issue should include which of the following?

The entry to record the bond issue would be: Cash 49,000 Bond discount 1,000 Bonds payable 50,000

Sun Corp. had investments in equity securities classified as trading costing $650,000. On June 30 of the current year, Sun decided to hold the investments indefinitely and accordingly reclassified them from trading to available-for-sale on that date. The investment's fair value was $575,000 at December 31 of the previous year; $530,000 at this June 30; and $490,000 at December 31 of the current year. What amount of loss from investments should Sun report in its current-year income statement?

The losses on the investment previous to this year have already been recognized in earnings. The losses from this year up until the reclassification from trading to available for sale (from $575,000 at the end of last year down to $530,000 when the reclassification was made) of $45,000 are recognized as a loss in earnings this year. Any loss after that goes into other comprehensive income.

Young Co. issues $800,000 of 10% bonds dated January 1, Year 1. Interest is payable semiannually on June 30 and December 31. The bonds mature in five years. The current market for similar bonds is 8%. The entire issue is sold on the date of issue. The following values are given: Present Value of Ordinary Annuity Present Value of $1 ---------------- ------------------- N=10; i=0.04 8.11090 0.67556 N=10; i=0.05 7.72173 0.61391 What amount of proceeds on the sale of bonds should Young report?

This question is about the computation of the issue price for a bond. The bonds will pay semiannually, and thus will pay $40,000 twice each year, computed as follows: Face amount of $800,000 × 10% coupon × 6/12 (half-year) = $40,000 The yield of the bonds is 8% annually, but in half-year periods it is 4% a half-year. The present value of the bonds is thus the $40,000 multiplied by the present value of the ordinary annuity for 10 periods and 4%, plus the $800,000 par value of the bonds multiplied by the present value of $1 at 10 periods, 4%: Issue price = ($40,000 × 8.11090) + ($800,000 × 0.67556) = $324,436 + $540,448 = $864,884

On December 31, 20X1, Byte Co. had capitalized software costs of $600,000 with an economic life of four years. Sales for 20X2 were 10% of expected total sales of the software. At December 31, 20X2, the software had a net realizable value of $480,000. In its December 31, 20X2, balance sheet, what amount should Byte report as net capitalized cost of computer software?

When software costs are capitalized, yearly amortization of these costs is based on the greater of the ratio of current sales to expected total sales or the straight-line method over the useful life of the asset (four years). Sales ratio: 10% (0.10) × $600,000 = $60,000 Straight-line: 25% (0.25) × $600,000 = $150,000 Since straight-line amortization is larger and is used, the remaining capitalized cost is $600,000 less $150,000, or $450,000. Since the net realizable value, $480,000, is greater than the $450,000, there is no need for an additional write-off.

An investor purchased a bond classified as a long-term investment between interest dates at a discount. At the purchase date, the carrying amount of the bond is more than:

neither the cash paid to the seller nor the face amount of the bond

An investor uses the cost method to account for an investment in common stock classified as an available-for-sale security. Dividends received this year exceeded the investor's share of investee's undistributed earnings since the date of investment. The amount of dividend revenue that should be reported in the investor's income statement for this year would be:

the portion of the dividends received this year that were not in excess of the investor's share of investee's undistributed earnings since the date of investment.

When purchasing a bond, the present value of the bond's expected net future cash inflows discounted at the market rate of interest provides what information about the bond?

Price

The discount resulting from the determination of a note payable's present value should be reported on the balance sheet as:

A note payable is sold at a discount when it is issued for less than face value. This would be recorded by the issuer as follows: Dr. Cr. Cash or other asset XX Discount or Note payable X Note Payable XXX This note would be reported on the balance sheet as follows: Note Payable XXX Less: Discount X --- Notes Payable (net of discount) XX The discount is reported as a direct reduction from the face amount of the note.

Bee Co. uses the direct write-off method to account for uncollectible accounts receivable. During an accounting period, Bee's cash collections from customers equal sales adjusted for the addition or deduction of the following amounts:

Accounts written off: deduction; Increase in accounts receivable balance: deduction

Delta, Inc., sells to wholesalers on terms of 2/15, net 30. Delta has no cash sales but 50% of Delta's customers take advantage of the discount. Delta uses the gross method of recording sales and trade receivables. An analysis of Delta's trade receivable balances on December 31, 20X1, revealed the following: Age Amount Collectible ------------ -------- ------------- 0 - 15 days $100,000 100% 16 - 30 days 60,000 95% 31 - 60 days 5,000 90% Over 60 days 2,500 $500 -------- $167,500 ======== On its December 31, 20X1, balance sheet, what amount should Delta report for the allowance for discounts?

Only receivables in the "0-15 days" age category are eligible for the cash discount (2/15, net 30) on December 31, 20X1. Allowance = Amount x Percent taking discount x Discount rate for discounts = $100,000 x 50% x 2% = $1,000

Gray Co. was granted a patent on January 2, 20X1, and appropriately capitalized $45,000 of related costs. Gray was amortizing the patent over its estimated useful life of 15 years. No impairment losses were recognized. During 20X4, Gray paid $15,000 in legal costs in successfully defending an attempted infringement of the patent. After the legal action was completed, Gray sold the patent to the plaintiff for $75,000. Gray's policy is to take no amortization in the year of disposal. In its 20X4 income statement, what amount should Gray report as gain from sale of patent?

Patents are carried at the purchase price or legal filing costs (if internally generated) and are amortized over the shorter of legal or useful life. Legal costs of successful patent defense are added to the patent carrying cost. Initial capitalized amount $45,000 amortization (3/15 x $45,000) (9,000) Infringement defense costs 15,000 -------- Carrying value at time of sale $51,000 ======== Sales price $75,000 Carrying value - 51,000 --------- Gain on sale $24,000 An intangible asset, other than goodwill, should be amortized over its useful life (if the life is finite) and should also be reviewed for impairment.

On October 1, Year 2, Park Co. purchased 200 of the $1,000 face amount, 10% bonds of Ott, Inc., for $220,000, including accrued interest of $5,000. The bonds, which mature on January 1, Year 9, pay interest semiannually on January 1 and July 1. Park used the straight-line method of amortization and appropriately recorded the bonds as a long-term investment. On Park's December 31, Year 3, balance sheet, the bonds should be reported at:

Since the purchase price was $220,000 and it included the accrued interest of $5,000, the price for the bonds was $215,000 ($220,000 - $5,000). The bonds were thus sold at a premium of $15,000 (the price of $215,000 less the face of $200,000, computed as 200 × $1,000). Applying straight-line, the premium will be amortized equally over the months remaining in the bond's term. The bond was bought on October 1 of Year 2, and will mature on January 1 of Year 9. That is a remaining term to maturity of 75 months (3 months of Year 2 and all of Years 3, 4, 5, 6, 7, and 8 ((6 × 12) + 3 = 75 months)). The total premium of $15,000 divided equally by 75 months is $200 per month. Thus, the bond carrying amount at December 31, Year 3, after the bond is held for 15 months (3 months in Year 2, and all of Year 3) will be: $215,000 (Initial price) − ($200 × 15 months) = $215,000 - $3,000 = $212,000

Universe Co. issued 500,000 shares of common stock in the current year. Universe declared a 30% stock dividend. The market value was $50 per share, the par value was $10, and the average issue price was $30 per share. By what amount will Universe decrease stockholders' equity for the dividend?

Stock dividends are accounted for by reclassifying a portion of retained earnings as contributed capital. They do not reduce assets or increase liabilities. Therefore, total stockholders' equity is not changed.

On April 1, 20X1, Hyde Corp., a newly formed company, had the following stock issued and outstanding: Common stock, no par, $1 stated value, 20,000 shares originally issued for $30 per share Preferred stock, $10 par value, 6,000 shares originally issued for $50 per share Hyde's April 1, 20X1, statement of stockholders' equity should report:

Summary journal entries for issuance of stock: Dr. Cr. (1) Common stock -- Cash ($30 x 20,000 shares) 600,000 Common stock ($1 x 20,000 shares) 20,000 Additional paid-in capital 580,000 (2) Preferred stock -- Cash ($50 x 6,000 shares) 300,000 Preferred stock ($10 x 6,000 shares) 60,000 Additional paid-in capital 240,000 At April 1, 20X1: Common stock = $20,000 Preferred stock = $60,000 Additional paid-in capital = $580,000 + $240,000 = $820,000

On January 1, a company issued a $50,000 face value, 8% 5-year bond for $46,139 that will yield 10%. Interest is payable on June 30 and December 31. What is the bond carrying amount on December 31 of the current year?

The bond carrying amount on December 31 of the current year is $46,768: Initial carrying value $46,139 Discount amortization at 6/30: Interest expense ($46,139 x .10 x 1/2) $2,307 Cash payment ($50,000 x .08 x 1/2) 2,000 307 ------- Carrying value at 7/1 $46,446 Discount amortization at 12/31: Interest expense ($46,446 x .10 x 1/2) $2,322 Cash payment ($50,000 x .08 x 1/2) 2,000 322 ------- Carrying value at 12/31 $46,768

In its 20X2 financial statements, Cris Co. reported interest expense of $85,000 in its income statement and cash paid for interest of $68,000 in its cash flow statement. There was no prepaid interest or interest capitalization either at the beginning or end of 20X2. Accrued interest on December 31, 20X1, was $15,000. What amount should Cris report as accrued interest payable in its December 31, 20X2, balance sheet?

The interest payable on January 1 was last year's expense, but was paid this year. Only $53,000 of this year's expense was paid this year, leaving $32,000. Interest expense for 20X2 $85,000 Less: Interest paid in 20X2 - 68,000 ------- Increase in accrued interest $17,000 Add: December 31, 20X1, accrued balance 15,000 ------- Accrued interest on December 31, 20X2 $32,000 =======

When the allowance method of recognizing uncollectible accounts is used, the entry to record the write-off of a specific account:

decreases both accounts receivable and the allowance for uncollectible accounts.

On July 31, 2015, Dome Co. issued $1,000,000 of 10%, 15-year bonds at par and used a portion of the proceeds to call its 600 outstanding 11%, $1,000 face value bonds, due on July 31, 2024, at 102. On that date, unamortized bond premium relating to the 11% bonds was $65,000. In its 2015 income statement, what amount should Dome report as gain or loss, before income taxes, from retirement of bonds?

Upon the retirement of a bond, the difference between the reacquisition price (what you pay) and the carrying amount of the bond is a gain or loss. Carrying value of 11% bonds $600,000 + $65,000 = $665,000 Less: Call price 600,000 x 102% = 612,000 Pretax gain from retirement of bonds $ 53,000

Carr, Inc., purchased equipment for $100,000 on January 1, Year 1. The equipment had an estimated 10-year useful life and a $15,000 salvage value. Carr uses the 200% declining balance depreciation method. In its Year 2 income statement, what amount should Carr report as depreciation expense for the equipment?

Year 1 depreciation = $100,000 × ((100% ÷ 10) × 2) = $20,000 Year 2 depreciation = ($100,000 - $20,000) × 0.20 = $16,000

When the allowance method of recognizing uncollectible accounts is used, the entries at the time of collection of a small account previously written off would:

increase the allowance for uncollectible accounts.

A 15-year bond was issued in Year 1 at a discount. During Year 11, a 10-year bond was issued at face amount with the proceeds used to retire the 15-year bond at its face amount. The net effect of the Year 11 bond transactions was to increase long-term liabilities by the excess of the 10-year bond's face amount over the 15-year bond's:

carrying amount.

Pal Corp.'s 20X1 dividend income included only part of the dividend received from its Ima Corp. investment. The balance of the dividend reduced Pal's carrying amount for its Ima investment. This reflects that Pal accounts for its Ima investment by the:

cost method, and only a portion of Ima's 20X1 dividends represent earnings after Pal's acquisition.

Marr Co. had the following sales and accounts receivable balances, prior to any adjustments at year-end: Credit Sales $10,000,000 Accounts Receivable 3,000,000 Allowance for uncollectible accounts 50,000 Marr uses 3% of accounts receivable to determine its allowance for uncollectible accounts at year-end. By what amount should Marr adjust its allowance for uncollectible accounts at year-end?

$40,000

A company recently acquired a copyright that now has a remaining legal life of 30 years. The copyright initially had a 38-year useful life assigned to it. An analysis of market trends and consumer habits indicated that the copyrighted material will generate positive cash flows for approximately 25 years. What is the remaining useful life, if any, over which the company can amortize the copyright for accounting purposes?

25 years

On June 30, Huff Corp. issued at 99, 1,000 of its 8%, $1,000 bonds. The bonds were issued through an underwriter to whom Huff paid bond issue costs of $35,000. On June 30, Huff should report the bond liability at:

Accounting Standards Update (ASU) 2015-03 requires that debt issuance costs be presented in the balance sheet as a direct deduction from the carrying amount of the related debt liability, consistent with debt discounts; the recognition and measurement guidance for debt issuance costs were not affected by the amendments. Amortization of debt issuance costs also shall be reported as interest expense; issue costs will no longer be reported in the balance sheet as deferred charges. The carrying value of the debt, initially, the bond liability, is $990,000, computed as the number of bonds multiplied by the face amount per bond, multiplied by the issue percentage, reduced by the bond issue costs of $35,000: 1,000 bonds × $1,000 face × 0.99 = $990,000 $990,000 − $35,000 = $955,000

Wilk Co. reported the following liabilities at December 31, 20X1: Accounts payable-trade $ 750,000 Short-term borrowings 400,000 Bank loan (current portion $100,000) 3,500,000 Other bank loan, matures June 30, 20X2 1,000,000 The bank loan of $3,500,000 was in violation of the loan agreement. The creditor had not waived the rights for the loan. What amount should Wilk report as current liabilities at December 31, 20X1?

Current liabilities represent obligations whose liquidation is expected to require the use of current assets or the creation of other current liabilities. Current liabilities also include long-term obligations that are or will be callable by the creditor because the debtor has violated a covenant in the debt agreement. The "other bank loan" matures in 6 months from the balance sheet date and is a current liability.

The controller of Peabody, Inc., has been asked to present an analysis of accounts receivable collections at the upcoming staff meeting. The following information is used: 12/31, Year 2 12/31, Year 1 Accounts receivable $100,000 $130,000 Allowance, doubtful accounts (20,000) (40,000) Sales 400,000 200,000 Cost of goods sold 350,000 200,000 What is the receivables turnover ratio as of December 31, Year 2?

For Year 2, sales were $400,000. To get average receivables, one needs to get the net beginning and net ending receivables balances, add them, and then divide the total by 2. Beginning balance was $130,000 - $40,000, or $90,000. Ending balance was $100,000 - $20,000, or $80,000. The average balance is $85,000: ($80,000 + $90,000) = $170,000; $170,000 ÷ 2 = $85,000. The receivables turnover is thus 4.7: $400,000 ÷ $85,000 = 4.7.

Quick Co. acquired the following assets from a liquidating competitor for a $200,000 lump-sum purchase price: Competitor's Carrying Amount Fair Value Inventory $ 70,000 $ 50,000 Land 40,000 50,000 Building 110,000 150,000 $220,000 $250,000 What amount should Quick report as the cost of the building?

GAAP requires that plant and equipment be recorded at historical cost, including the costs incurred necessary to bring them to the condition and location necessary for their intended use. For lump-sum acquisitions, the cost is allocated to the individual assets based upon their relative fair values. The cost of the building is $120,000, computed as follows: $150,000 ÷ $250,000 = 60%; $200,000 purchase price × 60% = $120,000.

Under IFRS, which of the following statements about intangible assets is correct?

Internally generated goodwill cannot be recognized as an asset.

On December 1, 20X1, Money Co. gave Home Co. a $200,000, 11% loan. Money paid proceeds of $194,000 after the deduction of a $6,000 nonrefundable loan origination fee. Principal and interest are due in 60 monthly installments of $4,310, beginning January 1, 20X2. The repayments yield an effective interest rate of 11% at a present value of $200,000 and 12.4% at a present value of $194,000. What amount of income from this loan should Money report in its 20X1 income statement?

Net proceeds of the loan were $194,000 and the effective interest rate was 12.4%. The journal entry to record the December 31, 20X1, accrual of interest would be: Debit Accrued Interest Receivable 2,005 Credit Interest Income 2,005 (12.4% x $194,000 x 1/12)

Gar Co. factored its receivables without recourse with Ross Bank. Gar received cash as a result of this transaction. How is this procedure best described?

Sale of Gar's accounts receivable to Ross, with the risk of uncollectible accounts transferred to Ross

A company issued rights to its existing shareholders without consideration. The rights allowed the recipients to purchase unissued common stock for an amount in excess of par value. When the rights are issued, which of the following accounts will be increased?

The issuance of rights was "without consideration" so no asset can be debited. This issuance should be recorded as a memo entry only. If, and when, the recipients exercise their rights at a later date, cash would be increased as well as common stock and additional paid-in capital. For now, however, none of the accounts would be increased.

The functional currency of Nash, Inc.'s, subsidiary is the euro. Nash borrowed euros as a partial hedge of its investment in the subsidiary. In preparing consolidated financial statements, Nash's translation loss on its investment in the subsidiary exceeded its exchange gain on the borrowing. How should the effects of the loss and gain be reported in Nash's consolidated financial statements?

The translation loss less the exchange gain is reported in other comprehensive income.

For a troubled debt restructuring involving only a modification of terms, which of the following items specified by the new terms would be compared to the carrying amount of the debt to determine if the debtor should report a gain on restructuring?

This question relates to the debtor's gain on troubled debt restructuring. FASB ASC 310-40-40-1 has changed the treatment of creditor's losses on a restructuring to include the use of present values. Debtor's gains, however, continue to follow FASB ASC 470-60-35-6. Debtor's gains are calculated based on undiscounted amounts. The total future cash payments, including interest, are used to compute the gain on troubled debt restructuring.

A fixed asset with a 5-year estimated useful life and no residual value is sold at the end of the second year of its useful life. How would using the sum-of-the-years'-digits method of depreciation instead of the double-declining-balance method of depreciation affect a gain or loss on the sale of the fixed asset?

To answer this question it is best to use an example and see how the numbers work out. If an asset with a 5-year life has a $100,000 cost and a zero salvage value, then the double-declining balance depreciation for the first year would be $40,000: $100,000 × 2/5 (twice straight-line) = $40,000 For the second year, depreciation would be $24,000: $100,000 - $40,000 = $60,000 $60,000 × 2/5 = $24,000 Thus, under double-declining balance, the remaining book value of the asset would be $36,000: $100,000 - $40,000 - $24,000 = $36,000 Under the sum-of-the-years'-digits method, depreciation for the first two years would be 5/15 (5/(5 + 4 + 3 + 2 + 1)) of $100,000, and then 4/15 of $100,000, for a total depreciation for the first two years of 9/15 × $100,000, or $60,000, leaving a book value of $40,000 under the sum-of-the-years'-digits depreciation method. With a higher book value under sum-of-the-years'-digits depreciation at the end of the second year, a sale at a loss would be a higher loss under sum-of-the-years'-digits method, and a sale at a gain would be a lower gain under the sum-of-the-years'-digits method.

On December 31, 20X1, Paxton Co. had a note payable due on August 1, 20X2. On January 20, 20X2, Paxton signed a financing agreement to borrow the balance of the note payable from a lending institution to refinance the note. The agreement does not expire within one year, and no violation of any provision in the financing agreement exists. On February 1, 20X2, Paxton was informed by its financial advisor that the lender is not expected to be financially capable of honoring the agreement. Paxton's financial statements were issued on March 31, 20X2. How should Paxton classify the note on its balance sheet at December 31, 20X1?

As a current liability because the lender is not expected to be financially capable of honoring the agreement

Jole Co. lent $10,000 to a major supplier in exchange for a noninterest-bearing note due in three years and a contract to purchase a fixed amount of merchandise from the supplier at a 10% discount from prevailing market prices over the next three years. The market rate for a note of this type is 10%. On issuing the note, Jole should record:

Discount on note receivable: Yes; Prepaid asset: Yes

Which of the following statements concerning the acquisition of assets is false?

Donated assets should be recorded at book value along with any incidental costs incurred.

Which inventory costing method would a company that wishes to maximize profits in a period of rising prices use?

FIFO

On August 15 of the current year, Benet Co. sold goods for which it received a note bearing the market rate of interest on that date. The 4-month note was dated this July 15. Note principal, together with all interest, is due November 15. When the note was recorded on August 15, which of the following accounts increased?

Interest receivable

During the year, Pitt Corp. incurred costs to develop and produce a routine, low-risk computer software product, as follows: Design of tools, jigs, molds, and dies involving new technology $125,000 Completion of detail program design 13,000 Costs incurred for coding and testing to establish technological feasibility 10,000 Other coding costs after establishment of technological feasibility 24,000 Other testing costs after establishment of technological feasibility 20,000 Costs of producing product masters for training materials 15,000 Duplication of computer software and training materials from product masters (1,000 units) 25,000 Packaging product (500 units) 9,000 In Pitt's December 31 balance sheet, what amount should be capitalized as software cost, subject to amortization?

The capitalized costs here are generally the costs incurred after attaining technological feasibility. Hence, the coding and testing costs of $24,000 and $20,000 are capitalized, but also the costs of the product masters for training of $15,000. Thus, a total of $59,000 would be capitalized, and the rest would be expensed as research and development.

House Publishers offered a contest in which the winner would receive $1,000,000, payable over 20 years. On December 31, 20X1, House announced the winner of the contest and signed a note payable to the winner for $1,000,000, payable in $50,000 installments every January 2. Also on December 31, 20X1, House purchased an annuity for $418,250 to provide the $950,000 prize monies remaining after the first $50,000 installment, which was paid on January 2, 20X2. In its December 31, 20X1, balance sheet, what amount should House report as note payable to contest winner, net of current portion?

The noncurrent portion of the note payable to contest winner should be reported on the balance sheet net of discount. This means that the present value of the 19 future payments of $50,000 is the correct amount to be disclosed. This amount is the $418,250 cost of the annuity.

A note receivable bearing a reasonable interest rate is sold to a bank with recourse. At the date of the discounting transaction, the notes receivable discounted account should be:

increased by the face amount of the note.

The replacement cost of an inventory item accounted for under the retail method is below the net realizable value.The inventory item's original cost is above the net realizable value. Under the lower of cost or market method, the inventory item should be valued at:

replacement cost.

The effective interest rate for a loan restructured in a troubled debt restructuring is based on:

the original contractual rate.

Anchor Co. owns 40% of Main Co.'s common stock outstanding and 75% of Main's noncumulative preferred stock outstanding. Anchor exercises significant influence over Main's operations. During the current period, Main declared dividends of $200,000 on its common stock and $100,000 on its noncumulative preferred stock. What amount of dividend income should Anchor report on its income statement for the current period related to its investment in Main?

An entity that exerts significant influence over another company in which it owns stock must use the equity method to account for its investment. Under this method, dividends received from an investee reduce the carrying amount of the investment but are not included in the income of the investor. However, an investment in preferred stock does result in dividend income. Consequently, Anchor will report dividend income of $100,000 × 0.75 ($75,000).

On December 31, 20X1, Largo, Inc., had a $750,000 note payable outstanding, due July 31, 20X2. Largo borrowed the money to finance construction of a new plant. Largo planned to refinance the note by issuing long-term bonds. Because Largo temporarily had excess cash, it prepaid $250,000 of the note on January 12, 20X2. In February 20X2, Largo completed a $1,500,000 bond offering. Largo will use the bond offering proceeds to repay the note payable at its maturity and to pay construction costs during 20X2. On March 3, 20X2, Largo issued its 20X1 financial statements. What amount of the note payable should Largo include in the current liabilities section of its December 31, 20X1, balance sheet?

Based on this requirement, Largo, Inc., should exclude $500,000 ($750,000 less the $250,000 prepayment) of the note payable—the amount refinanced—from current liabilities. Thus, $250,000 of the note payable, which was paid on January 12, 20X2, would be included in current liabilities on December 31, 20X1.

A company has a parcel of land to be used for a future production facility. The company applies the revaluation model under IFRS to this class of assets. In Year 1, the company acquired the land for $100,000. At the end of Year 1, the carrying amount was reduced to $90,000, which represented the fair value at that date. At the end of Year 2, the land was revalued, and the fair value increased to $105,000. How should the company account for the Year 2 change in fair value?

By recognizing $10,000 in profit or loss and $5,000 in other comprehensive income

On July 1, Year 1, Kay Corp. sold equipment to Mando Co. for $100,000. Kay accepted a 10% note receivable for the entire sales price. This note is payable in two equal installments of $50,000 plus accrued interest on December 31, Year 1 and Year 2. On July 1, Year 2, Kay discounted the note at a bank at an interest rate of 12%. Kay's proceeds from the discounted note were:

By the time of the discounting, some of the principal has already been paid. Only the second installment, the final $50,000 principal plus interest, will be paid to the bank when due. At the end of December, Year 2, the $50,000 will be received by the bank along with 10% interest (since the principal will have been outstanding for a whole year). On December 31, Year 2, a total of $55,000 maturity value will be due: $50,000 + ($50,000 × 0.1) = $55,000 The discounted proceeds will be based on this amount, the discount rate (0.12), and the discounting period (from July to December of Year 2, 6 months). The discount amount is thus: $55,000 × 0.12 × 6/12 = $3,300 The cash proceeds are the maturity value less the discount: $55,000 - $3,300 = $51,70

Which of the following ratios useful in assessing the liquidity position of a company?

Defensive-interval ratio, a measure of time the company can survive (continue to pay operating expenses in cash) using only the quick assets (cash, marketable securities, and net accounts receivable). Thus, it is computed by dividing total quick assets by average daily cash expenditures.

On August 1, 20X1, Vann Corp.'s $500,000, 1-year, noninterest-bearing note due July 31, 20X2, was discounted at Homestead Bank at 10.8%. Vann uses the straight-line method of amortizing discount. What amount should Vann report for notes payable in its December 31, 20X1, balance sheet?

Determine the discount amount (the amount of interest on the note). The face amount of the note less the discount is the initial proceeds, the initial carrying value of the note. The discount is interest, accrued equally (straight-line) over 12 months, and added to the carrying value of the note for the five months to the end of the year. Discount on note = 10.8% x $500,000 = $54,000 Monthly amortization = $54,000 / 12 months = $ 4,500/month Face amount of note $500,000 Less discount at issuance 54,000 -------- Carrying value of note at issuance $446,000 Add discount amortization Aug. 1 - Dec. 31 (5 months x $4,500) 22,500 -------- Carrying value of note on December 31, 20X1 $468,500 ========

Album Co. issued 10-year, $200,000 debenture bonds on January 2. The bonds pay interest semiannually. Album uses the effective interest method to amortize bond premiums and discounts. The carrying value of the bonds on January 2 was $185,953. A journal entry was recorded for the first interest payment on June 30, debiting interest expense for $13,016 and crediting cash for $12,000. What is the effective interest rate for the debenture bonds?

Effective interest = Carrying value of the bonds × Effective interest rate × Time period In this case: $13,016 = $185,953 × Unknown effective interest rate × 1/2 year Effective interest rate = ($13,016 ÷ $185,953) × 2 = 14%

Regal Department Store sells gift certificates, redeemable for store merchandise, that expire one year after their issuance. Regal has the following information pertaining to its gift certificates sales and redemptions: Unredeemed on December 31, 20X1 $ 75,000 20X2 sales 250,000 20X2 redemptions of prior-year sales 25,000 20X2 redemptions of current-year sales 175,000 Regal's experience indicates that 10% of gift certificates sold will not be redeemed. In its December 31, 20X2, balance sheet, what amount should Regal report as unearned revenue?

Estimate the total number of certificates to be redeemed prior to expiration and then subtract those already used up. 20X2 Sales of gift certificates $250,000 Less estimated nonredemption 10% x $250,000 = 25,000 -------- Net redeemable certificates 225,000 Less 20X2 redemptions of current-year sales 175,000 -------- Unearned revenue on December 31, 20X2 $ 50,000 ========

Case Cereal Co. frequently distributes coupons to promote new products. On October 1, 20X1, Case mailed 1,000,000 coupons for $.45 off each box of cereal purchased. Case expects 120,000 of these coupons to be redeemed before the December 31, 20X1, expiration date. It takes 30 days from the redemption date for Case to receive the coupons from the retailers. Case reimburses the retailers an additional $.05 for each coupon redeemed. As of December 31, 20X1, Case had paid retailers $25,000 related to these coupons and had 50,000 coupons on hand that had not been processed for payment. What amount should Case report as a liability for coupons in its December 31, 20X1, balance sheet?

Even though the coupons expired December 31, 20X1, additional coupons can be expected after December 31, 20X1, since the problem states that it takes 30 days from the redemption date for Case to receive the coupons from the retailer. Therefore, the entire estimated redemption of 120,000 should be used to accrue the estimated liability even though only 100,000 coupons have been accounted for on December 31, 20X1 (50,000 coupons have been redeemed and 50,000 are awaiting processing). Expected liability Coupons expected Cost of for 20X1 = to be redeemed x redemption = 120,000 x ($.45 + $.05) = $60,000 Liability on December 31, 20X1 = Expected liability - Payments made = $60,000 - $25,000 = $35,000

Puff Co. acquired 40% of Straw, Inc.'s, voting common stock on January 2, 20X1, for $400,000. The carrying amount of Straw's net assets at the purchase date totaled $900,000. Fair values equaled carrying amounts for all items except equipment, for which fair values exceeded carrying amounts by $100,000. The equipment has a 5-year life. During 20X1, Straw reported net income of $150,000. What amount of income from this investment should Puff report in its 20X1 income statement?

FASB ASC 323-10-05-5 requires the use of the equity method when a company acquires 20% or more of the outstanding stock of another company. Significant influence is assumed. Under the equity method, Puff should report 40% of the $150,000 income of Straw, or $60,000. Because Straw's equipment has a fair market value exceeding its carrying value, Puff should amortize the difference over the equipment's 5-year life. Puff should record 40% of $100,000 ($40,000) as equipment subject to amortization (depreciation). Straight-line amortization of $40,000 over five years yields an expense of $8,000. Puff has income of $60,000 less $8,000, or $52,000 for 20X1.

Verona Co. had $500,000 in short-term liabilities at the end of the current year. Verona issued $400,000 of common stock subsequent to the end of the year, but before the financial statements were issued. The proceeds from the stock issue were intended to be used to pay the short-term debt. What amount should Verona report as a short-term liability on its balance sheet at the end of the current year?

FASB ASC 470-10-45-14 requires that short-term obligations be reported as long-term liabilities if a company (1) intends to refinance the short-term obligation on a long-term basis and (2) demonstrates the ability to refinance it a long-term basis. The intent is stated in the problem. Verona's issuance of common stock for $400,000 before the statements were issued demonstrates the ability to refinance $400,000 of the short-term obligations on a long-term basis. The balance of the obligation ($100,000) must be reported as a current liability.

Cole Co. began constructing a building for its own use in January 20X1. During 20X1, Cole incurred interest of $50,000 on specific construction debt, and $20,000 on other borrowings. Interest computed on the weighted-average amount of accumulated expenditures for the building during 20X1 was $40,000. What amount of interest cost should Cole capitalize?

For qualifying assets being constructed for an entity's own use, FASB ASC 835-20-30-2 requires interest cost to be capitalized equal to the less of (a) the avoidable interest (based on the weighted-average amount of accumulated expenditures), or (b) the actual interest cost incurred. Cole's avoidable interest is given to be $40,000. Since the $70,000 actual interest cost incurred ($50,000 + $20,000) is greater than the avoidable interest of $40,000, the amount of interest that Cole can capitalize is $40,000.

In 20X1, May Corp. acquired land by paying $75,000 down and signing a note with a maturity value of $1,000,000. On the note's due date, December 31, 20X6, May owed $40,000 of accrued interest and $1,000,000 principal on the note. May was in financial difficulty and was unable to make any payments. May and the bank agreed to amend the note as follows: The $40,000 of interest due on December 31, 20X6, was forgiven. The principal of the note was reduced from $1,000,000 to $950,000 and the maturity date extended one year to December 31, 20X7. May would be required to make one interest payment totaling $30,000 on December 31, 20X7. As a result of the troubled debt restructuring, May should report a gain, before taxes, in its 20X6 income statement of:

Gain on troubled debt restructuring for May Corp. in 20X6 is computed as follows: Gain = Total due on debt - Required restructured payment = Principal + Accrued interest - (Restructured principal + Required interest) = ($1,000,000 + $40,000) - ($950,000 + $30,000) = $1,040,000 - $980,000 = $60,000

On July 1, Year 1, Pell Co. purchased Green Corp. 10-year, 8% bonds with a face amount of $500,000 for $420,000. The bonds mature on June 30, Year 9, and pay interest semiannually on June 30 and December 31. Using the interest method, Pell recorded bond discount amortization of $1,800 for the six months ended December 31, Year 1. From this long-term investment, Pell should report Year 1 revenue of:

If bonds are purchased at a discount, then the discount is immediately recorded as a credit in the acquiring corporation's books. As the discount is amortized, it is thus debited, to decrease it. When cash is received as interest, the additional debit to amortize the discount adds to the debit to cash to increase the total credit to recognized revenue. Therefore, the total revenue for the year will be the cash received as interest over the semiannual period, $500,000 (face amount) × 0.08 (8%) × 6/12, or $20,000, plus the $1,800 discount amortized, for a total revenue for the year of $21,800.

At the acquisition date, July 2, 20X1, reporting unit R has a fair value of $370,000 and a carrying amount (including goodwill of $100,000) of $470,000. On December 31, 20X1, the fair value of the assets and liabilities assigned to reporting unit R is $330,000, and the fair value of R is $400,000. The goodwill impairment loss reportable is:

Impairment of goodwill is a two-step process: Step 1, Compare: (a) year-end fair value of reporting unit $400,000 (b) carrying amount, including goodwill $470,000 If (b) exceeds (a), go to step 2. If (a) exceeds (b), no impairment. Step 2, Compare: (a) implied fair value of reporting unit goodwill ($400,000 - $330,000) $ 70,000 (b) carrying amount of goodwill $100,000 Since (b) exceeds (a) by $30,000, an impairment loss of $30,000 is recognized. If (a) exceeds (b), no impairment.

On incorporation, Dee, Inc., issued common stock at a price in excess of its par value. No other stock transactions occurred, except treasury stock was acquired for an amount exceeding this issue price. If Dee uses the par value method of accounting for treasury stock appropriate for retired stock, what is the effect of the acquisition on the following?

Journal entry for acquisition of treasury stock using par value method: Dr. Cr. Treasury stock (common stock) XXX Additional paid-in capital XX Retained earnings X Cash XXXX The debit to Retained Earnings is for the excess of reacquisition cost over original issue price.

Which of the following statements is correct regarding the provision for income taxes in the financial statements of a sole proprietorship?

No provision for income taxes is required.

On December 31, 20X1, Jet Co. received two $10,000 notes receivable from customers in exchange for services rendered. On both notes, interest is calculated on the outstanding principal balance at the annual rate of 3% and payable at maturity. The note from Hart Corp. made under customary trade terms, is due in nine months and the note from Maxx, Inc., is due in five years. The market interest rate for similar notes on December 31, 20X1, was 8%. The compound interest factors to convert future values into present values at 8% follow: Present value of $1 due in nine months: .9440 Present value of $1 due in five years: .6806 At what amounts should these two notes receivable be reported in Jet's December 31, 20X1, balance sheet?

The Hart note would be reported at its face amount of $10,000 since it matures within the current 1-year accounting period. The correct value for reporting the Maxx note is: Present value of Maxx note = Maturity amount x Present value factor = ($10,000 + ($10,000 x 3% x 5 years)) x .6806 = ($10,000 + $1,500) x .6806 = $7,827

A company manufactures and distributes replacement parts for various industries. As of December 31, Year 1, the following amounts pertain to the company's inventory: Net Cost to Normal Replacement Sale Sell or Profit Item Cost Cost Price Dispose Margin ---------- ------- ----------- -------- ------- ------- Blades $41,000 $ 38,000 $ 50,000 $ 2,000 $15,000 Towers 52,000 40,000 54,000 4,000 14,000 Generators 20,000 24,000 30,000 2,000 6,000 Gearboxes 80,000 105,000 120,000 12,000 8,000 What is the total carrying value of the company's inventory as of December 31, Year 1, under IFRS?

The International Financial Reporting Standards (IFRS) apply a lower of cost and net realizable value, applied on an item-by-item basis. For each item, one must figure out the net realizable value: the sales price less cost to sell. For blades, net realizable value is $50,000 less $2,000, or $48,000. For blades the cost is less, so blades will be carried at cost, $41,000. Towers sell for $54,000 less $4,000 to sell, for a net realizable value of $50,000, which is below cost of $52,000. Towers are carried at $50,000. Generators sell for $30,000 less $2,000 for a net realizable value of $28,000, and cost of $20,000 is below that. Generators will be carried at cost, $20,000. Gearboxes sell for $120,000 less $12,000, or a net realizable value of $108,000, which is higher than cost of $80,000. Gearboxes will be carried at cost of $80,000. The total of all four items at lower of cost or net realizable value is thus $41,000 + $50,000 + $20,000 + $80,000 = $191,000.

On April 1, 20X2, Hill Corp. issued 200 of its $1,000 face value bonds at 101 plus accrued interest. The bonds were dated November 1, 20X1, and bear interest at an annual rate of 9% payable semiannually on November 1 and May 1. What amount did Hill receive from the bond issuance?

The bonds themselves were sold for 101% of face value, but they were issued between interest payment dates, and thus five months of accrued interest was also received, and will be paid back (along with another month's interest) at the scheduled interest payment date. Sales price of bonds = 1.01 x 200 x $1,000 = $202,000 Accrued interest = (5/12) (.09 x 200 x $1,000) = 7,500 -------- Total received from bond issuance $209,500 ========

On January 2, 20X1, West Co. issued 9% bonds in the amount of $500,000, which mature on January 2, 20X8. The bonds were issued for $475,658 to yield 10%. Interest is payable annually on December 31. West uses the interest method of amortizing bond discount. In its June 30, 20X1, balance sheet, what amount should West report as bonds payable?

The bonds were issued at a discount, and one must amortize the discount up to the balance sheet date. The interest paid is 9% of the face amount of $500,000, but the interest expense is the yield multiplied by the carrying amount. The difference of these is the discount amortization added to the carrying amount of the bonds. Face amount of bonds $500,000 Less: Issue price 475,658 Bond discount at issue date $ 24,342 June 30, 20X1, journal entry to accrue interest and amortize discount: Dr. Cr. Bond interest expense 23,783 (10% x $475,658 x 6/12) Discount on bonds payable ($23,783 - $22,500) 1,283 Interest Payable (9% x $500,000 x 6/12) 22,500 Bonds payable on June 30, 20X1, balance sheet: Bonds payable $500,000 Less: Unamortized discount ($24,342 - $1,283) 23,059 Net bonds payable $476,941

On January 1, Year 1, Fox Corp. issued 1,000 of its 10%, $1,000 bonds for $1,040,000. These bonds were to mature on January 1, Year 11, but were callable at 101 any time after December 31, Year 4. Interest was payable semiannually on July 1 and January 1. On July 1, Year 6, Fox called all of the bonds and retired them. Bond premium was amortized on a straight-line basis. Before income taxes, Fox's gain or loss in Year 6 on this early extinguishment of debt was:

The bonds were issued at a premium of $40,000 ($1,040,000 - $1,000,000). The premium is amortized using straight-line, over the term of the bonds, $40,000 ÷ 10 years (from January of Year 1 to January of Year 11), or $4,000 premium amortized each year. The bonds were called on July 1, Year 6, for $1,010,000, the call price (1,000 bonds × $1,000 per bond × 1.01 call percentage). By July 1, Year 6, 5-1/2 years have gone by from the issuance on January 1, Year 1. Thus, the remaining unamortized premium on the bonds is the initial total of $40,000 - (5.5 years × $4,000), or $18,000 ($40,000 - $22,000). The call price of the bonds was $1,010,000 and the carrying value of the bonds was $1,018,000 ($1,000,000 + $18,000), so the debt was paid for less than the carrying amount, and a gain of the $8,000 difference is recognized ($1,018,000 - $1,010,000).

Young Corp. purchased equipment by making a down payment of $4,000 and issuing a note payable for $18,000. A payment of $6,000 is to be made at the end of each year for three years. The applicable rate of interest is 8%. The present value of an ordinary annuity factor for three years at 8% is 2.58, and the present value for the future amount of a single sum of one dollar for three years at 8% is 0.79. Shipping charges for the equipment were $2,000, and installation charges were $3,500. What is the capitalized cost of the equipment?

The capitalized cost of the equipment is $24,980: Down payment $ 4,000 Present value of note ($6,000 x 2.58) 15,480 Shipping charges 2,000 Installation charges 3,500 ------- Total $24,980

Webb Co. has outstanding a 7%, 10-year $100,000 face-value bond. The bond was originally sold to yield 6% annual interest. Webb uses the effective interest rate method to amortize bond premium. On June 30, Year 2, the carrying amount of the outstanding bond was $105,000. What amount of unamortized premium on bond should Webb report in its June 30, Year 3, balance sheet?

The interest paid for the year from June 30, Year 2. to June 30, Year 3. is based on the face amount ($100,000) multiplied by the stated 7% payment rate: $100,000 × 0.07 = $7,000 The interest expense using the interest method is based on the carrying amount of the debt multiplied by the yield of the debt: $105,000 × 0.06 = $6,300 The premium amortized from June 30, Year 2, to June 30, Year 3, is the difference of these two amounts: $7,000 - $6,300 = $700 Thus, the premium of $5,000 on June 30, Year 2, ($105,000 - $100,000, carrying amount less face amount) is lowered by the $700 premium amortization down to $4,300: $5,000 - $700 = $4,300

Kew Co.'s accounts payable balance at December 31 of the prior year was $2,200,000 before considering the following data: Goods shipped to Kew FOB shipping point on December 22 of the prior year were lost in transit. The invoice cost of $40,000 was not recorded by Kew. On January 7 of the current year, Kew filed a $40,000 claim against the common carrier. On December 27 of the prior year, a vendor authorized Kew to return, for full credit, goods shipped and billed at $70,000 on December 3 of the prior year. The returned goods were shipped by Kew on December 28 of the prior year. A $70,000 credit memo was received and recorded by Kew on January 5 of the current year. Goods shipped to Kew FOB destination on December 20 of the prior year, were received on January 6 of the current year. The invoice cost was $50,000. What amount should Kew report as accounts payable in its prior-year December 31 balance sheet?

The items shipped to the company on December 22 shipping point should be added to inventory, even though the items were lost. The title to the goods transferred at the time the common carrier picked them up and the goods were part of the company inventory upon receipt by the common carrier. (The common carrier owes the company the price of the goods lost in transit, but that is a separate legal matter.) The items that were returned to the vendor in the prior year (with the vendor's approval) should be taken out of purchases and removed from the payables for the year. The goods shipped FOB destination that did not arrive until January were not the company inventory until the title passed in January (upon arrival), so they should not be added to payables. Thus, the end-of-year payables should be $2,170,000: $2,200,000 + $40,000 - $70,000 = $2,170,000

Brock Co. adopted the dollar-value LIFO inventory method as of January 1, 20X1. A single inventory pool and an internally computed price index are used to compute Brock's LIFO inventory layers. Information about Brock's dollar-value inventory follows: INVENTORY At Base At Current At Dollar Date Year Cost Year Cost Value LIFO ---------- --------- --------- ---------- January 1, 20X1 $40,000 $40,000 $40,000 20X1 layer 5,000 14,000 6,000 --------- --------- --------- December 31, 20X1 45,000 54,000 46,000 20X2 layer 15,000 26,000 ? --------- --------- --------- December 31, 20X2 $60,000 $80,000 ? ========= ========= ========= What was Brock's dollar value LIFO inventory on December 31, 20X2?

To add an increase, a new layer, using dollar-value LIFO (last in, first out), two items are needed: the total real increase (computed using base-year costs) and the total increase in price level to date so far. Once both of these amounts are acquired, multiply them and add the product to the beginning-year inventory total. 20X2 index = Inventory at current-year cost / Inventory at base-year cost = $80,000 / $60,000 = 1.333 20X2 layer = 20X2 index x 20X2 layer at base-year cost = 1.333 x $15,000 = $20,000 Dollar value LIFO inventory on December 31, 20X2 = 12/31/X1 valuation + 20X2 layer = $46,000 + $20,000 = $66,000

On January 2 of the current year, Otto Co. purchased 40% of Penn Co.'s outstanding common stock. The carrying amount of Penn's depreciable assets was $1,000,000 on January 2. Penn's depreciable assets had an original useful life of 10 years, and a remaining useful life of 5 years. Otto recognized $8,000 amortization for the current year ending December 31 related to its investment in Penn due to the excess of fair value over book value on these assets. What was the fair value of Penn's depreciable assets on January 2 of the current year?

When a purchase is made of the stock of another company, and the equity method is applicable to it (generally from 20% to 50% ownership acquired), if the depreciable assets have a fair value in excess of the book value, then the acquiring corporation may need to use its purchase price (the higher fair value) for those assets to figure out its depreciation when recognizing its earnings from the equity method holdings. The owned corporation, Penn, continues to depreciate its assets at their book value, but the owning corporation, Otto, will use the fair value of those assets at the time of purchase to figure out income. Thus, Otto must adjust its income (from Penn) as Penn's income less the additional depreciation, which is part of the purchase price. The additional depreciation ($8,000) is computed as the total fair value of the assets multiplied by the percentage of ownership (40%), divided by the remaining useful life of the assets. Working backwards: $8,000 × 5 years (Remaining useful life) ÷ 0.40 (Ownership) = $100,000 Thus, when Otto bought in, the fair value of Penn must have been $100,000 over book value, or $1,100,000.

During January 20X1, Metro Co., which maintains a perpetual inventory system, recorded the following information pertaining to its inventory: Unit Total Units Units Cost Cost on Hand ----- ------ ------ ------- Balance (January 1, 20X1) 1,000 $1 $1,000 1,000 Purchased (January 7, 20X1) 600 3 1,800 1,600 Sold (January 20, 20X1) 900 700 Purchased (January 25, 20X1) 400 5 2,000 1,100 Under the LIFO method, what amount should Metro report as inventory on January 31, 20X1?

When applying a LIFO (last in, first out) perpetual system, the sales are taken first from the purchase immediately preceding the sale, then the purchase before that (all the way back to beginning inventory, in that order). The ending inventory here is thus the remaining part of the beginning inventory, and the most recent purchase. Balance on January 1, 20X1 (1,000 units at $1.00) $1,000 Purchase on January 7, 20X1 (600 units at $3.00) + 1,800 -------- LIFO inventory on January 7, 20X1 $2,800 Sale of 900 units on January 20, 20X1 (600 units at $3.00) - 1,800 (300 units at $1.00) - 300 -------- LIFO inventory on January 20, 20X1 (700 units at $1.00) $ 700 Purchase on January 25, 20X1 (400 units at $5.00) + 2,000 -------- LIFO inventory on January 31, 20X1 (1,100 units) $2,700 ========

Walt Co. adopted the dollar-value LIFO inventory method as of January 1, 20X1, when its inventory was valued at $500,000. Walt's entire inventory constitutes a single pool. Using a relevant price index of 1.10, Walt determined that its December 31, 20X1, inventory was $577,500 at current-year cost, and $525,000 at base-year cost. What was Walt's dollar-value LIFO inventory on December 31, 20X1?

When applying dollar-value LIFO (last in, first out), one must first compute the ending inventory total using base-year prices, to tell how much the total inventory (at base-year prices) has increased or decreased. If this is an increase, then add the increase amount (multiplied by the year's price level) to the beginning inventory total. $ 525,000 December 31, 20X1, inventory at base-year cost - $500,000 January 1, 20X1, inventory at base-year cost ---------- $25,000 20X1 layer at base-year cost x 1.10 20X1 index ---------- $27,500 20X1 layer at December 31, 20X1, cost ========== Dollar-value LIFO inventory on December 31, 20X1 Base-year layer $500,000 20X1 layer at 20X1 cost 27,500 -------- Total $527,500 ========

Taylor Corp., which began operations in 20X1, accounts for revenues using the installment method. Taylor's sales and collections for the year were $60,000 and $35,000, respectively. Uncollectible accounts receivable of $5,000 were written off during 20X1. Taylor's gross profit rate is 30%. In its December 31, 20X1, balance sheet, what amount should Taylor report as deferred gross profit?

When applying the installment sales method, annual records have to be maintained for sales, cost of goods sold, and remaining receivables for each year (kept separately). Each year's gross profit percentage is maintained, and the total year's gross profit is recognized as cash is collected for the receivables. Deferred gross profits are the year's receivables multiplied by the year's gross profit percentage. Sales for 20X1 $60,000 Less: Collections in 20X1 $35,000 Accounts written off in 20X1 5,000 40,000 ------- ------- Uncollected Sales on 12/31/X1 $20,000 Times gross profit rate x 0.30 ------- Equals deferred gross profit on 12/31/X1 $ 6,000 =======

Moss Corp. owns 20% of Dubro Corp.'s preferred stock and 80% of its common stock. Dubro's stock outstanding on December 31, 20X1, is as follows: 10% cumulative preferred stock $100,000 Common stock 700,000 Dubro reported net income of $60,000 for the year ending December 31, 20X1. What amount should Moss record as equity in earnings of Dubro for the year ending December 31, 20X1?

When computing the income share for an investor, first deduct the cumulative preferred dividends from net income. Moss' share of preferred dividends = 20% x 10% x $100,000 = $2,000 Earnings attributable to common shareholders = $60,000 - 10% x $100,000 = $50,000 Moss' share of common earnings = 80% x $50,000 = $40,000 Moss' total equity in Dubro earnings = $2,000 + $40,000 = $42,000 FASB ASC 323-10-35-16 states: "If an investee has outstanding cumulative preferred stock, an investor shall compute its share of earnings (losses) after deducting the investee's preferred dividends, whether or not such dividends are declared." These amounts may be adjusted later into consolidated statements.

On December 30, 20X1, Hale Corp. paid $400,000 cash and issued 80,000 shares of its $1 par value common stock to its unsecured creditors on a pro rata basis pursuant to a reorganization plan under Chapter 11 of the bankruptcy statutes. Hale owed these unsecured creditors a total of $1,200,000. Hale's common stock was trading at $1.25 per share on December 30, 20X1. As a result of this transaction, Hale's total stockholder's equity had a net increase of:

When debts are paid with stock and property, the difference of the fair value of the stock and property and the carrying value of the debt (if less than the carrying amount of the debt) is gain. Gain is added to retained earnings, increasing equity, and the stock issued adds to equity as well. Amount owed to unsecured creditors $1,200,000 Less cash paid $400,000 Common stock issued at fair value* (80,000 shares x $1.25) 100,000 500,000 -------- --------- Gain on restructuring $ 700,000 ========= * Common stock would be increased by $80,000 (80,000 sh x $1 par). Additional Paid-in Capital would be increased by $20,000 (80,000 sh x ($1.25 - $1)). The net increase in Hale Corp.'s stockholders' equity is $800,000, the sum of the $700,000 gain on restructuring plus the $100,000 increase in stockholders' equity resulting from issuance of the additional shares.

On January 2, 20X1, Nast Co. issued 8% bonds with a face amount of $1,000,000 that mature on January 2, 20X7. The bonds were issued to yield 12%, resulting in a discount of $150,000. Nast incorrectly used the straight-line method instead of the effective interest method to amortize the discount. How is the carrying amount of the bonds affected by the error?

When the bond's yield requires a discount, the bond's interest expense is based (early on) on a lower principal, and thus the expense applying straight-line would be higher (it is an average expense for the term). The discount amortization would be too high for the first year (overstating bond carrying value), but under both methods at the end of the term, the carrying amounts will be the bond face amount. Carrying amount on 1/2/X1 = $1,000,000 - $150,000 = $850,000 Amortization of discount: Using straight-line = $150,000 / 6 yrs = $25,000 / yr. Using effective interest = (0.12 x $850,000) - (.08 x $1,000,000) = $22,000 Carrying amount on 12/31/X1: Using straight-line = $850,000 + $25,000 = $875,000 Using effective interest = $850,000 + $22,000 = $872,000 Overstatement of carrying value when using straight-line = $875,000 - $872,000 = $3,000 Over the 6-year life of the bonds, the same total discount amortization will occur under each method. The bond carrying amount on January 2, 20X7, will be the maturity value regardless of the amortization method. Thus, on December 31, 20X1, the bond carrying value will be overstated if straight-line amortization of discount is used but on January 2, 20X7, there will be no effect from its use.

The following data pertain to Thorne Corp. for the current calendar year: Net income $240,000 Dividends paid on common stock 120,000 Common stock outstanding (unchanged during year) 300,000 shares The market price per share of Thorne's common stock at December 31 was $12. The price-earnings ratio at December 31 was:

You are correct, the answer is C. The price-to-earnings ratio is the relationship between the stock price per share to the earnings per share. The stock price per share is given as $12, but the earnings per share will have to be computed. Earnings per share is net income divided by common shares outstanding: $240,000 ÷ $300,000 = 0.8 Thus, the price-to-earnings ratio is $12 ÷ 0.8, or 15 to 1.


Related study sets

Ch 24 Urinary System, Ch 25 Fluid + Electrolyte Balance

View Set

legal environment of business chapter 5 parker

View Set

Chapter 11: Nursing Management: Patients With Chronic Obstructive Pulmonary Disease and Asthma

View Set

Parts of Speech: Words and Basic Phrases

View Set

Publication Manual of the American Psychological Association , Section 1.16 Planning for Ethical Compliance and Chapter 6

View Set

MKTG 4120 Chapter 13 Review Questions

View Set

Chapter 32: The Building of Global Empires

View Set